Register | Sign In


Understanding through Discussion


EvC Forum active members: 65 (9162 total)
9 online now:
Newest Member: popoi
Post Volume: Total: 915,815 Year: 3,072/9,624 Month: 917/1,588 Week: 100/223 Day: 11/17 Hour: 0/0


Thread  Details

Email This Thread
Newer Topic | Older Topic
  
Author Topic:   A Critique of the "Evolution Essay" A GREAT DEBATE S1WC and anglagard ONLY
anglagard
Member (Idle past 836 days)
Posts: 2339
From: Socorro, New Mexico USA
Joined: 03-18-2006


Message 49 of 100 (353523)
10-01-2006 7:28 PM


Genetic Relationships and Bad Sources
From where I left off:
S1WC writes:
“In fact, it has even been suggested that apes evolved from man!” [18]
Your source indicates Duane Gish is the one responsible for this statement. Since humans are apes (no tail for one thing), what is this supposed to mean?
Skip that one as a point I say, do you disagree?
Point 41:
How strange! But there are other ideas to which evolutionists hold on to, like similarities between apes and man.
Evolutionists say that apes and humans are very similar, this is supposed to support their belief that apes evolved into humans. Let us look at this idea. Henry Morris wrote,
“Many evolutionists have tried to argue that humans are 99% similar chemically to apes and blood precipitation tests do indicate that the chimpanzee is people’s closest relative. Yet regarding this we must observe the following: “Milk chemistry indicates that the donkey is man’s closest relative.” “Cholesterol level tests indicate that the garter snake is man’s closest relative.” “Tear enzyme chemistry indicates that the chicken is man’s closest relative.” “On the basis of another type of blood chemistry test, the butter bean is man’s closest relative.”” [19]
So, who are we really related to? The monkey, the donkey, the chicken, the garter snake, or the butter bean? Hmmm? How about non of those?
By the monkey, do you mean chimpanzee? For the umpteenth time Chimpanzees are not monkeys, for one thing, no tail.
As to the assertion concerning human and donkey milk:
quote:
We have not found a direct comparison of human and chimpanzee milk chemistry. R. E. Sloan, et al., showed that human milk proteins (whey and casein) were much more like macaque milk than donkey milk (Comparative Biochemistry and Physiology, 1961, 4:47-62). Human and chimpanzee milk lysozymes are identical. Even this limited comparison disproves the creationist claim that the donkey is our nearest relative based on milk chemistry.
Concerning chicken tear enzymes:
quote:
The enzyme referred to here is lysozyme, which is found in human milk, tears, leukocytes, and so forth. Variants exist in tissues of other species, for example, in chicken egg whites. Prager and Wilson showed that chicken lysozyme differs from human lysozyme by fifty-one out of 130 amino acids (in E. F. Osserman, Lysozyme, Academic Press, 1974, pp. 127-141). Chimpanzee lysozyme is identical to human lysozyme. It is apparent that the creationists either had not bothered to look at this paper when they made their claims or they believe that fifty-one is less than zero.
Concerning garter snake cholesterol levels:
quote:
Cholesterol is a simple lipid (a wax) and its structure doesn't vary among species. Furthermore, its concentration can vary several hundredfold in an individual human depending upon diet and genetic background. Therefore, it is a useless molecule for determining genetic similarity. This datum isn't just wrong, it's nonexistent.
Concerning butter bean blood chemistry:
quote:
Blood antigen A. This is one of the molecules that determine blood types. They are called glycoproteins because they have sugars attached to a protein. Butterbeans contain a sugar configuration that is similar enough to the glycoprotein sugar that it can react with antibodies directed against the A blood type if the butterbean sugar is at a high concentration (Gottschalk, Glycoproteins, 1972). Chimpanzees have blood antigens that are identical or nearly identical to those of humans (J. Ruffie, "Immunogenetics of Primates" in Perspectives in Primate Biology edited by A. B. Chigrelli, Plenum Press, 1972, p. 217). Butterbeans, having no blood, obviously have no blood antigens.
The above refutations are courtesy the National Center for Science Education at http://www.ncseweb.org/...07_volume_3_number_1__3_4_2003.asp
Don’t you feel a bit silly quoting the crackpot Morris as a source?
Point 42:
Another thought - if we are 99% similar to apes, why do we rule over them as if they were lower than us by so much? I mean, if the ape is so related to us, how come we as humans have so much power over them? Why do we act like kings over all animals, if the ape was our great, great ancestor? Shouldn’t the ape have great power if it’s descendants a long way down are humans, the ones with power over the whole animal kingdom? When did the great power feature evolve? We as humans have morality, a characteristic no monkey or any other animal has. We also have intelligence, a complex spoken language, and ethical values. [20] So how is this? When humans evolved, morality evolved as well?!? Intelligence? A very complex spoken language? Ethical values? The ability to read and write? Excellent comprehension? Great reasoning skills? Mathematical comprehension? Everything else?
I am not sure what you intend by this. Of course humans have greater power than other species, primarily due to science and its derivative, technology, how does this disprove evolution? Some of these attributes, such as ethical values and morality are subjective human-centered opinions that have no empirically valid cross-species application. For example, power over others, human or not, is not necessarily moral or ethical in application.
From the eminently quotable Henry Morris:
quote:
Yet the prophecy again has its obverse side. Somehow they have only gone so far and no farther. The Japhethites and Semites have, sooner or later, taken over their territories, and their inventions, and then developed them and utilized them for their own enlargement. Often the Hamites, especially the Negroes, have become actual personal servants or even slaves to the others. Possessed of a genetic character concerned mainly with mundane matters, they have eventually been displaced by the intellectual and philosophical acumen of the Japhethites and the religious zeal of the Semites.
Don’t you feel a bit dirty quoting the crackpot Morris as a source?
To be fair, the source, Richard Trott, does not consider Morris a racist, just a buffoon, see the source at Creationism Implies Racism?
How’s that for an example of quote mining? Perhaps with enough practice I could get as good at it as Morris and Gish.
Also, there is a lot of human variance in such qualities as “Intelligence? A very complex spoken language? Ethical values? The ability to read and write? Excellent comprehension? Great reasoning skills? Mathematical comprehension?” Some counterexamples among individuals are readily available over the internet, as can be seen above.
ABE - Upon reflection, I guess you are arguing that because of civilization, humans can't be related to other life forms and therefore evolution is not true. I would be cautious in using that argument because the same scientific methodology that currently sustains modern society, that same ability in math, reasoning, and comprehension, is also used by the biosciences and geosciences to support evolution.
As an aside, there are arguments that the sciences should be diminished in areas where they conflict with particular views of a minority of self-proclaimed personally inerrent interpreters of various religions, primarily in Christian fundamentalist belief systems in the US.
The problem with any attempt to diminish or eliminate broad areas of the sciences is that such an action would automatically diminish any technology dependent upon scientific advancement for its productivity and efficiency. Such an action would condemn the economy, military, education, and health system of any given nation that adopted such an attack upon selected sciences to permanant second-rate status in all fields mentioned above.
At that point, such a given nation would easily fall prey to other political forces that would have a less anagonistic view of science. A scenario that would be considered "evolution" by the inevitable victors.
Point 43:
Yet another fact, apes have 48 chromosomes, but humans have 46 chromosomes. This is a very big difference, and a very important one. Chromosomes play a big role in animals and humans. Also, aside from all this, there are many other differences between apes and humans, too many to count!
Let’s see how big that difference in chromosomes is, shall we?
From Wikipedia Chimpanzee Genome Project article at: Chimpanzee genome project - Wikipedia
quote:
Human and common chimpanzee chromosomes are very similar. The primary difference is that humans have one fewer pair of chromosomes than do other great apes. In the human evolutionary lineage, two ancestral ape chromosomes fused at their telomeres producing human chromosome two. There are only nine other major chromosomal differences between chimpanzees and humans: chromosome segment inversions on human chromosomes 1, 4, 5, 9, 12, 15, 16, 17, 18. After the completion of the Human genome project, a Common Chimpanzee genome project was initiated. In December of 2003, a preliminary analysis of 7600 genes shared between the two genomes confirmed that certain genes such as the forkhead-box P2 transcription factor, which is involved in speech development, have undergone rapid evolution in the human lineage. Several genes involved in hearing were also found to have changed rapidly during human evolution, suggesting selection involving human language-related behavior. Differences between individual humans and Common Chimpanzees are about 10 times the typical difference between pairs of humans.
Apparently, some of these differences between chimpanzees and humans have been directly traced to specific genes. Wonder what the future of genetic analysis may hold for the assertions of Morris and Gish?
Remember the chromosome difference? Here is what the article says:
quote:
The results of the chimpanzee genome project suggest that when ancestral chromosomes 2A and 2B fused to produce human chromosome 2, no genes were lost from the fused ends of 2A and 2B. At the site of fusion, there is approximately 150,000 base pairs of sequence not found in chimpanzee chromosomes 2A and 2B. Additional linked copies of the PGML/FOXD/CBWD genes exist elsewhere in the human genome, particularly near the p end of chromosome 9. This suggests that a copy of these genes may have been added to the end of the ancestral 2A or 2B prior to the fusion event. It remains to be determined if these inserted genes confer a selective advantage.
” PGML. The phosphoglucomutase-like gene of human chromosome 2. This gene is incomplete and may not produce a functional transcript [6].
” FOXD. The forkhead box D4-like gene is an example of an intronless gene. The function of this gene is not known, but it may code for a transcription control protein.
” CBWD. Cobalamin synthetase is a bacterial enzyme that makes vitamin B12. In the distant past, a common ancestor to mice and apes incorporated a copy of a cobalamin synthetase gene (see: Horizontal gene transfer). Humans are unusual in that they have several copies of cobalamin synthetase-like genes, including the one on chromosome 2. It remains to be determined what the function of these human cobalamin synthetase-like genes is. If these genes are involved in vitamin B12 metabolism, this could be relevant to human evolution. A major change in human development is greater post-natal brain growth than is observed in other apes. Vitamin B12 is important for brain development, and vitamin B12 deficiency during brain development results in severe neurological defects in human children.
” CXYorf1-like protein. Several transcripts of unknown function corresponding to this region have been isolated. This region is also present in the closely related chromosome 9p terminal region that contains copies of the PGML/FOXD/CBWD genes.
” Many ribosomal protein L23a pseudogenes are scattered through the human genome.
For those who prefer, here is that picture:
It appears to me that not only are chimpanzees and humans related, scientists are now honing in on just how related, exactly where related, and exactly what sequence of chemicals results in what makes us related (and different as well).
Heaven forbid they may use this evil science to cure diseases or make life more comfortable!
So what have Gish and Morris (or more accurately, the legacy of Morris), contributed to civilization lately?
The change from ape to man is too great to have ever occurred. Even more so, for a single cell to evolve into humans! But similarities between apes and man are not the only similarities that evolutionists bring up.
Give me a week. Have done enough damage already.
Edited by anglagard, : ABE in point 42 and correct example of passive voice

Replies to this message:
 Message 56 by Someone who cares, posted 10-17-2006 12:30 AM anglagard has not replied

  
anglagard
Member (Idle past 836 days)
Posts: 2339
From: Socorro, New Mexico USA
Joined: 03-18-2006


Message 50 of 100 (353544)
10-02-2006 12:29 AM
Reply to: Message 47 by Someone who cares
09-29-2006 8:51 PM


Message Concerning RAZD GD
S1WC writes concerning RAZD and Great Debate possibility:
But I feel he likes to attack me/my words in particular. Could be that it's just his way of posting, but he always likes to attack me/my words when I post. Like when I came back here for round two of debating, RAZD was suspended. Right after my post, he was de-suspended and already replying to me. I feel something strange in this. I don't know how the suspending/de-suspending process works, but it seems to me like he de-suspended himself just to reply to me or something.
I do not remember the exact circumstances concerning any suspension, but I do know that RAZD is not an admin to my knowledge and therefore can't de-suspend himself.
As to the possibility of a GD, RAZD has replied in this thread http://EvC Forum: Definitions, Daffynitions, Delusions, Logic and Critical Thinking. -->EvC Forum: Definitions, Daffynitions, Delusions, Logic and Critical Thinking. as he is not supposed to reply in this one.
This is what he stated:
I'll debate SW1C - as said before - when he meets my precondition of changing the grossly erroneous misrepresentation regarding lucy and the knee fossil in his "essay" that has already been pointed out to him, ie - he demonstrates that he can debate in good faith and admit when he is just plain wrong
This is the message, which I deliver here as a matter of courtesy, since you have stated you do not read the other threads.

This message is a reply to:
 Message 47 by Someone who cares, posted 09-29-2006 8:51 PM Someone who cares has replied

Replies to this message:
 Message 57 by Someone who cares, posted 10-17-2006 12:41 AM anglagard has not replied

  
anglagard
Member (Idle past 836 days)
Posts: 2339
From: Socorro, New Mexico USA
Joined: 03-18-2006


Message 58 of 100 (356971)
10-17-2006 1:31 AM
Reply to: Message 53 by Someone who cares
10-06-2006 7:59 PM


Re: Real Busy
Just wanted to give you the opportunity to catch up. As said before, I intend to go through the entire essay before responding to your subsequent explanations.
I believe it is quite honorable of you to admit making some mistakes. Perhaps the next revision of your essay will see significant improvement as a result of this process. In the future, I intend to provide an essay concerning my theological beliefs in which all will be welcome to correct.
I understand you are busy with the business of living, as I often am. Understood, I will be patient and not race to the end before providing you an opportunity to respond.
Now the ball appears to be in my court, will proceed through the next few paragraphs soon.

This message is a reply to:
 Message 53 by Someone who cares, posted 10-06-2006 7:59 PM Someone who cares has replied

Replies to this message:
 Message 60 by Someone who cares, posted 10-19-2006 11:39 PM anglagard has replied

  
anglagard
Member (Idle past 836 days)
Posts: 2339
From: Socorro, New Mexico USA
Joined: 03-18-2006


Message 62 of 100 (359580)
10-29-2006 1:20 AM


Correcting Some Odd Misconceptions
Sorry, I’ve taken so long to respond but have been busy and it has taken me awhile to get over the shock that anyone would use the diversity and adaptability of life as an argument against evolution.
From where we left off:
Point 44:
Evolutionists also like to mention the fact that many animals have similar structures like lungs, stomachs, brains, bones, etc., and they use this as support for their theory of common ancestry. But let us reason. If animals are similar, wouldn’t the belief that they had one Creator account for this better? A Creator who used a similar pattern when creating all the animals? One Creator, one basic pattern? Isn’t that a much better explanation? Let’s look at it this way. Why do all houses have roofs? Or walls? Or other parts? Because those parts work well in protection from harsh weather, from insects, from rain pouring in, etc. Same here. God used similar patterns in creation because they are most practical, and they work well.
Here is an example of similar patterns, also known as parallel evolution, of placental and marsupial mammals. Here is a picture from Encyclopedia Britannica showing such similarities.
Why would a proposed micromanaging creator not settle on one manner of giving birth between placental mammals and marsupials? There are major differences according to Natural History Collections: Mammals so you don’t just have to take my word for it.
quote:
Marsupials like opossums give birth to very immature young which develop attached to nipples in an abdominal pouch or 'marsupium'.
The rectum and urinogenital sinus open together at a common, short cloaca. The 266 species of living marsupials are confined to the Americas and Australiasia.
quote:
Placental mammals also give birth to live young but their young are nurtured before birth and develop to a relatively mature condition within a uterus. The foetuses are attached to the mother by an allantoic placenta. There is no cloaca and the anus opens separately from the urinogenital tract. There are about 3800 species of placental mammals. They occur in terrestrial and aquatic habitats throughout the world.
Is this not a refutation of all species having one perfect design for each purpose? Why are the differences related to geography? Why are some animals cold-blooded and some warm-blooded? Wouldn’t one or the other be the final decision of the divine micromanager? What about flying organisms? A bat has a leathery wing, a bird a feathery wing, and an insect a membranous wing. If there is a perfect wing why are there three types? Why do fish have bones but sharks have cartilage? Why do blindfish have eyes when in an embryonic stage? When an animal goes extinct, why are they not replaced by an exact copy of the formerly successful species if the design was perfect? Why was the Ichthyosaur replaced by the killer whale? I could go on an on.
Or, another explanation could be, the fact that we have one major environment and we have similar needs.
That other explanation is called evolution. That is why in many cases body designs of marsupials and placental mammals are similar, although the method of giving birth and the design of the birth canal differ.
I’d say point 44 refuted due to existence of marsupials alone.
Point 45:
Like we all live in basically one major environment with the same oxygen, so we have lungs to use that oxygen and keep our bodies alive.
When you refer to ”we’ in the above sentence, are you referring to all living things? Insects do not have lungs. Anaerobic bacteria do not use oxygen.
Or, we have water, salt and fresh kinds, so we have organs in our bodies that absorb the water and replenish our cells with it. Or we also have one sun, so plants have mechanisms and chlorophyll to turn solar energy into food. We have similar structures, because we have one basic environment, not because we had a common ancestor.
When you refer to one basic environment, are you referring to all environments that harbor life? From this article Sorry, that's a dead link (404) | Natural History Museum apparently eukaryotic life can thrive from below freezing to 70C, from a Ph of near 0 to 10, even from thermal vents in the bottom of the ocean. Is this all the one basic environment you refer to? How about bacteria that thrive two miles down in rock?
From: Otherworldly bacteria discovered two miles do | EurekAlert!
quote:
As Lin explained: "We know how isolated the bacteria have been because our analyses show that the water they live in is very old and hasn't been diluted by surface water. In addition, we found that the hydrocarbons in the local environment did not come from living organisms, as is usual, and that the source of the hydrogen (H2) needed for their respiration comes from the decomposition of water (H2O) by radioactive decay of uranium, thorium, and potassium."
Humans and most other land-dwelling organisms ultimately get their energy from the Sun, with photosynthetic plants forming the base of the food web. But in dark places where sunlight doesn't reach, life has to depend on other energy sources. Other communities of "chemoautotrophs"--a word chained together from Greek roots meaning "chemical self-nourishment"--have been found in exotic places such as aquifers, petroleum reservoirs, and vents linked to deep-sea volcanoes. Yet these communities all depend at least in part on nutrients that can be traced back to photosynthetic plants or bacteria.
Point 45 refuted, as seen above. Not all life depends upon breathing oxygen or photosynthesis.
Point 46:
But we do have various habitats and climates all over the world, so we have variation, to better adapt to those specific habitats and climates. Wouldn’t the belief that we were all created into our environment to suit our need for survival in it, be a better explanation than evolving into our environment, for the similarities we see?
Just considering humans why would a presumably infallible designer design such poorly performing structures as the human spine and knee. What happened to such perfect design that matches the human head with the birth canal, an obvious evolutionary compromise between large brain cases and hips designed for upright locomotion?
Point 47:
Because evolution is unguided, how would evolution make a creature know to evolve a stomach and teeth for the specific foods, and the right size of a mouth for the sizes of fruits and vegetables?
One is that animals do not pre-know how to evolve according to Darwinian evolution. Are you attacking Lamarckian evolution? Do you know the difference? Two is that the hand and mouth do not perfectly fit all fruits and vegetables. Do you mean those human hands and mouths that perfectly fit watermelons and cantaloupes?
Or hands the perfect size and shape to grasp fruits, vegetables, branches, etc? Or legs, with the right bones, muscles, ligaments, to allow us to walk on flat ground, rocky plains, or to climb a mountain, or a tree?
Interesting, are you claiming now that all fruits, vegetables, and tree branches are a perfect fit to human hands? Or are human hands an appropriately evolved compromise to manipulate most objects regardless of size?
Or lungs to utilize the oxygen in our environment? Or anything else? Similarities do not show having a common ancestor.
I would say that similarities obviously show having a common ancestor due to imperfection of design. The very existence of extinction and replacement, which has occurred in recorded history to dodos, passenger pigeons, and the Tasmanian wolf, among other species, clearly refutes the idea that each species is specifically and perfectly designed for its environment.
Point 47 refuted due to existence of melons, tree branch size variations, and extinction.
Point 48:
For example, the eye of a human is very similar to the eye of an octopus. Yet no one is claiming that they had a common ancestor!
Not true, any evolutionary biologist would state all life had a common ancestor, including octopi and humans. See Comparative Analysis of Gene Expression for Convergent Evolution of Camera Eye Between Octopus and Human by Atsushi Ogura, Kazuho Ikeo, and Takashi Gojobori in Genome Res. 2004 August; 14(8): 1555-1561.
The abstract:
quote:
Although the camera eye of the octopus is very similar to that of humans, phylogenetic and embryological analyses have suggested that their camera eyes have been acquired independently. It has been known as a typical example of convergent evolution. To study the molecular basis of convergent evolution of camera eyes, we conducted a comparative analysis of gene expression in octopus and human camera eyes. We sequenced 16,432 ESTs of the octopus eye, leading to 1052 nonredundant genes that have matches in the protein database. Comparing these 1052 genes with 13,303 already-known ESTs of the human eye, 729 (69.3%) genes were commonly expressed between the human and octopus eyes. On the contrary, when we compared octopus eye ESTs with human connective tissue ESTs, the expression similarity was quite low. To trace the evolutionary changes that are potentially responsible for camera eye formation, we also compared octopus-eye ESTs with the completed genome sequences of other organisms. We found that 1019 out of the 1052 genes had already existed at the common ancestor of bilateria, and 875 genes were conserved between humans and octopuses. It suggests that a larger number of conserved genes and their similar gene expression may be responsible for the convergent evolution of the camera eye.
Full text available here: Comparative Analysis of Gene Expression for Convergent Evolution of Camera Eye Between Octopus and Human
The above article not only shows that evolutionary biologists believe octopi and humans have a common ancestor but also how they independently evolved eyes from shared genes from that common ancestor. Point refuted.
Additionally, the human eye is not exactly the same as that of an octopus.
See Evolution: Change: Life's Grand Design
Here it shows the design of the human eye is not optimal relative to the octopus despite the common ancestry.
quote:
Visual quality is degraded because light scatters as it passes through several layers of cellular wiring before reaching the retina. Granted, this scattering has been minimized because the nerve cells are nearly transparent, but it cannot be eliminated because of the basic design flaw. Moreover, the effects are compounded because a network of vessels, which is needed to supply the nerve cells with a rich supply of blood, also sits directly in front of the light-sensitive layer, another feature that no engineer would propose.
A more serious flaw occurs because the neural wiring must poke directly through the wall of the retina to carry the nerve impulses produced by photoreceptor cells to the brain. The result is a blind spot in the retina -- a region where thousands of impulse-carrying cells have pushed the sensory cells aside. Each human retina has a blind spot roughly a millimeter in diameter -- one that would not exist if only the eye were designed with its sensory wiring behind rather than in front of the photoreceptors The optic nerve connects to the brain through a hole in the retina, causing a blind spot.
Do these design problems exist because it is impossible to construct an eye that is wired properly, so that the light-sensitive cells face the incoming image? Not at all. Many organisms have eyes in which the neural wiring is neatly tucked away behind the photoreceptor layer. The squid and the octopus, for example, have a lens-and-retina eye quite similar to our own, but their eyes are wired right-side out, with no light-scattering nerve cells or blood vessels in front of the photoreceptors, and no blind spot.
Why not?!?
Because from the above anyone can see it’s not true.
Point 48 refuted, simple misrepresentation of what evolutionary scientists believe.
Point 49:
So who decides when similarities show common ancestry and when they don’t?!?
Bioscientists and geoscientists with years of training decide based upon scientific principles such as morphology and cladistics. Most people who criticize evolution do so without the slightest knowledge of what evolution means, the terms used in evolution, the processes proposed by evolution, the tremendous evidence for evolution in several scientific disciplines, or indeed even what constitutes basic logic and critical thinking.
Instead we get clowns like Morris claiming butter beans have blood chemistry like humans when any kindergartener not enslaved by willful ignorance and self-delusion can tell you beans do not have blood.
Now, if we did evolve from one cell, why do we have all of this variation?
Evolution.
Why do we have so many different kinds of animals and plants?
Evolution.
Evolution is supposed to be unguided, random, and without a controlling outside force.
No it isn’t. The controlling force is adaptation to the environment through mutation, natural selection, gene flow, sexual selection, and genetic drift. For more details see: http://taxonomy.zoology.gla.ac.uk/...volution/l3/causes.html
How would the fish know to evolve, or start evolving legs to climb out onto land? What would make it know that legs would work well on land? Remember, there is no outside guiding force in evolution. Also, why would some organisms evolve into animals, while others into plants that are eaten by the animals? WHY? Why did some monkeys stay monkeys, while others evolved into intelligent humans who rule over the monkeys? But evolution is not just based upon similarities, evolution also has some mechanisms with which it is said to have worked and to be working now.
How do you know when to breathe or make your heart beat when asleep? How does a tree or bacteria know how to grow and reproduce without a brain? Does the divine micromanager intervene in each act of breathing, every involuntary function, and even every act of reproduction among each species?
Also, humans did not evolve from monkeys, they both evolved from a common ancestor.
Point 49 refuted due to obvious and complete misunderstanding of what the Theory of Evolution actually means.
Edited by AdminJar, : scale image

Replies to this message:
 Message 63 by anglagard, posted 10-29-2006 3:27 PM anglagard has not replied
 Message 65 by Someone who cares, posted 10-29-2006 10:49 PM anglagard has not replied
 Message 66 by Someone who cares, posted 10-30-2006 11:53 PM anglagard has replied

  
anglagard
Member (Idle past 836 days)
Posts: 2339
From: Socorro, New Mexico USA
Joined: 03-18-2006


Message 63 of 100 (359681)
10-29-2006 3:27 PM
Reply to: Message 62 by anglagard
10-29-2006 1:20 AM


Re: Correcting Some Odd Misconceptions
Still going.
Point 50:
One of such mechanisms of evolution is natural selection. Natural selection is the mechanism by which one kind of animal or plant is supposed to evolve into another one. There are some problems with this mechanism though. First of all, it is worth mentioning that evolutionists believe natural selection worked with mutations to make animals and plants evolve. Natural selection only gets rid of information and certain organisms, it doesn’t add information to the genetic code, or new organisms into the world.
One form of evolution in miniature, where positive mutations are created and selected for, is the immune response. When an unknown foreign virus invades the body, the occasional white blood cell contains a weak antibody that has some marginally deleterious effect on the viral invader. As the rampant viral invader creates an environment that selects for this marginal effect, the cells producing the weak antibody are under a great pressure to select for beneficial mutations of the weak antibody to increase the immune response. Eventually, through the process of mutation and natural selection, the cells evolve to produce stronger and stronger antibodies and the viral invader and the disease it causes are overcome.
From: The Evolution of Improved Fitness
quote:
Clearly what we observe in the antibody response is evolution in miniature. In this model we can learn the structure of each antibody gene at the beginning of the experiment before the challenge with antigen, and observe the accumulation of randomly induced mutations under natural selection for progressively improved function. This model of evolution is similar to the computer simulation discussed earlier, but it has three advantages as a persuasive example. First, it is a natural biological phenomenon rather than a theoretical designed simulation. Second, the initial unmutated sequence is selected for its ability to bind the antigen molecule, so it clearly has some function (which must be "undesigned," since the antigen may be a novel synthetic chemical never found in nature). In contrast, critics of Dawkins's weasel model argue that it is unreasonable to assume that the initial random sequence chosen by the computer could have any function at all. Third and finally, as in real phylogenetic evolution, the selection pressure is for biological function rather than for a specific target sequence chosen by an intelligent "creator." Thus the different sets of mutations observed in different high affinity antibodies that bind the same antigen represent alternative solutions to a particular selective challenge, just as different globin sequences in different species represent alternative solutions to the need for an oxygen-carrying protein.
Obviously there are differences between this kind of antibody evolution and the phylogenetic evolution that produced the diversity of plants and animals that we find on our planet. But none of these differences critically weaken the logic of the analogy between these two kinds of evolution as examples of random mutation and selection. Both involve sequences altered by random mutations, including rare beneficial alterations that "take over" the population because of their increased efficiency in proliferating under selective pressure; then these mutants are themselves "taken over" by later mutations, leading to progressively more efficient structures.
Thus the molecular immunogenetics evidence of antibody evolution that I have described makes it clear that, contrary to the creationists' claims, the combination of random mutation and selection CAN be a potent creative biological engine for the generation of progressive functional improvements. This evidence alone does not prove that life evolved as Darwin suggested, but it highlights the emptiness of another invalid, though superficially appealing, creationist objection to evolution: the false idea that random mutation is a uniformly deleterious process that could never be the source of improved biological function. And, to people who can appreciate the amazing complexity of life as a thing of wonder, the story of the generation of antibody diversity reveals in the immune system another example of an undesigned but beautifully functioning system.
Therefore, the immune system, which is present in all humans and higher order animals, is essentially an example of beneficial mutations acting under natural selection. You have an example of the very system of which you deny existence acting within your own body.
This also brings up another point, namely that the imposition of anti-evolution dogma in education, may, and indeed probably will, result in unnecessary sickness and death among the populace because the medical establishment would not know, or be allowed, to treat illness as it really works but rather according to the misinterpretation of ancient texts.
Point 50 refuted due to existence of immune system.
Point 51:
But that is exactly what we would need to get a single cell or something even simpler, to evolve into a human over time, great increases of information in the DNA. We do not see this with natural selection. Natural selection uses circular reasoning in its logic as applied to evolution. A species of animal or plant now existing has survived due to the fact that it was fit. Yet it has to be fit, because it has survived, as it can obviously be seen. In all animals and plants, we see that they reproduce after their own kind. Cows produce cows, parrots produce parrots, oak trees produce oak trees. This is what we observe, not one kind evolving into another one. Dogs produce dogs, and haven’t turned into another kind of animal, and they can’t! [21] Now even if mutations did work, they wouldn’t work like evolutionists claim they did for evolution of one kind to the next.
Of course to properly respond to this, one would have to define the term “kind,” something that has never been done in the history of this debate to my knowledge. However, if ”kind’ is defined as species here are some examples of historically observed speciation from Observed Instances of Speciation
Unfortunately the great length of this article prevents me from posting the relevant parts along with the evidence showing why they are considered true speciation events.
Here is the list of new species covered:
quote:
Evening Primrose (Oenothera gigas)
Kew Primrose (Primula kewensis)
Tragopogon
Raphanobrassica
Hemp Nettle (Galeopsis tetrahit)
Madia citrigracilis
Brassica
Maidenhair Fern (Adiantum pedatum)
Woodsia Fern (Woodsia abbeae)
Stephanomeira malheurensis
Maize (Zea mays)
Yellow Monkey Flower (Mimulus guttatus)
Drosophila paulistorum
Drosophila melanogaster (several speciation experiments using different scenarios)
Apple Maggot Fly (Rhagoletis pomonella)
Gall Former Fly (Eurosta solidaginis)
Flour Beetles (Tribolium castaneum)
Lab Rat Worm, Nereis acuminata
Chlorella vulgaris
If that is not enough here is some more from:
Evolving Before Our Eyes / Songbirds and salamanders bolster Darwin's theory that change in habitat can create 2 species from one
In the case of the Warblers:
quote:
Finally, in Siberia where the two populations of the warblers coexist at the northern end of their ring of habitats, the two varieties of Phylloscopus could no longer recognize each other's songs at all. In the warblers, mating songs are essential to sexual selection, and warbler songs were keys to their species evolution.
"Our results show how gradual divergence in a trait involved in mate choice leads to the formation of new species," Irwin wrote in the team's Nature report
The colored bars on the wings of the two groups of birds that coexist in Siberia also differed markedly, Irwin found. One group wears a single yellow bar on each wing, while the other bears two - another key to sexual selection, he says.
All those differences, and especially the fact that the two groups of birds in Siberia do not interbreed, provide the most defining evidence that the two songbird populations have become truly separate species, Irwin and Price agree.
"They act like separate species, and the genetic evidence supports that conclusion," Irwin said. "In central Siberia today, the original species has definitely become two species."
And in the case of the salamanders:
quote:
It is in the San Diego area that the original Ensatina species, with its shiny black body and mottled red legs, belly and tail, meets an even more vividly colored subspecies called E. klauberi, whose brilliant lemon-yellow body is striped in black from head to tail.
To Wake, the changes in the varied subspecies of Ensatina are another clear example of evolutionary changes toward "incipient species formation." On Mount Palomar, where two strikingly different populations of Ensatina have come together, "they are indeed two species," he noted in a recent interview.
"What Darren Irwin has found with his warbler ring species and what we have found with our Ensatina ring are two compelling pieces of evidence for true speciation," Wake concluded.
Of course, we are hung up once again on definitions, this time the definition of kind, so point 51 is in abeyance.
Such inability to define transitional or kind, or to use standard definitions, renders logic unusable and therefore rational arguments can’t be made. I can’t debate someone who uses a language like Kligon which I don’t know and they either refuse to teach or choose to use a nonstandard “made up” definition that renders all scholarship unusable.
Point 52:
Because mutations are merely tiny, usually harmful or neutral, random changes to the DNA code. They most often give off either a neutral effect, or a negative effect. They alter the DNA code just slightly. That is all mutations can do. Mutations are not capable of adding any new information to the genetic code, as would be needed to make a single cell evolve into a human over time. Mutations only alter the information that is already in the genetic code, they do not, and cannot, add any new information. Thus mutations would not make evolution possible. About 99.9% of mutations are neutral or harmful! They would only make the case worse for evolution by destroying the organism. Plus, mutations that actually have their effect on organisms are rare. But evolution would require almost an infinite amount of changes to produce man from a single cell, and mutations don’t even make the proper changes for evolution to occur! It is impossible! For example, an earthquake ruins a city, it doesn’t build it. But that is what evolutionists are believing in when they discuss mutations and natural selection. [22] But, even if one beneficial mutation were to occur, many hundreds of harmful one would follow it, overruling the beneficial one, and then leading the organism to its ultimate destruction. [23]
It must be tough to be a YEC having to deny the existence of the immune system in order to support anti-evolution dogma. To claim the immune system leads the host organism to its ultimate destruction ranks with that butter bean blood chemistry as an example of utterly delusional thinking (in my humble opinion).
Point 52 refuted due to existence of the immune system.
Point 53:
Also, mutations in the reproductive cells are the only ones to pass on to offspring. Any mutations in other body cells do not pass on to offspring. So even IF a monkey did learn to walk upright, its descendants wouldn’t walk upright. [24] Animals only reproduce after their own kind. The descendants can only take the information from their parents, the information that is embedded in their DNA from birth. They cannot change into a different kind, they already have their DNA code preprogrammed, and that’s how it will be for them, not in any different way. In the animals and plants, new varieties of the same kind are produced, not different kinds! [25] Most of us have probably heard of “survival of the fittest,” used when discussing about evolution. But “survival of the fittest” only shows us how an organism has survived, not how it has evolved! [26] We all know that the strongest animals of the pack survive. But that does not show evolution! The “urge” for an animal or plant to evolve does not exist anywhere in chemistry. [27]
Already addressed in Point 51.
Point 54:
The cells in our bodies need each other to survive, they are dependant upon one another. As with organs in a system. For example, in order for food to get from our mouths to our stomachs, we need a tube connecting them. Or to breathe, we need not only lungs, but also a nose or a mouth, and an expanding rib cage, and a diaphragm. But if the cells or organs were evolving, how could we have survived? Our body parts need interdependency to survive, but if they were at different stages of evolving, we wouldn’t be able to survive. How could a human or an animal survive with underdeveloped vital organs or systems, like the systems needed for eating, digesting food, breathing, excreting, sending impulses through the nervous system, etc.? We wouldn’t survive! Or how could mankind even reproduce, if the reproductive organs were still evolving? How could a bird eat and live, if the beak and stomach were still evolving? Besides, natural selection is supposed to only help evolve that which is necessary for our survival.
This is that strange concept of half-evolved organs that keeps popping up in the essay. Here is a rebuttal to the LaPointe essay from Mark I. Vuletic which makes similar claims concerning partially evolved organs at: Page not found
quote:
In other words LaPointe views the sequence of eye development in organisms in the following way:
1. 1/4 eye
2. 1/2 eye
3. 3/4 eye
4. fully formed eye
But this is a misrepresentation of the evolutionary development of the eye. Rather than having the eye appear in progressively larger fractions, the evolutionary account has the eye appear in progressively more complex forms:
1. Light-sensitive spot
2. Light-sensitive cup
3. Light-sensitive cup with pinhole opening
4. Light-sensitive cup with adjustable opening (iris)
5. Light-sensitive cup with iris and transparent covering (lens)
6. etc.
Each step of the progressive complexification of the eye confers additional survival ability upon the organism which possesses the structure, and it is not difficult to see how these steps could be bridged by simple mutations. Moreover, each step of the progressive complexification of the eye is represented in species which exist today.
Likewise, gradations exist for lungs, sex organs, wings, feathers, and just about every biological structure under the sun. These structures are not the all-or-nothing deals creationists frequently take them to be.
Here is a picture of the way it really works from Kathryn 'Katie' Klaene:
I think once again this shows you do not understand how evolution works and are arguing against a straw man of your own creation.
Point 54 refuted.
Point 55:
So how did we obtain social skills, love, care, feeling sorrow for others, friendship, and other feelings and emotions? How could those feelings and emotions, not necessary for survival, evolve! Socially, natural selection would require selfish behavior to survive. Meaning, only those who are strongest will, and should, live. It proclaims that those who have power and get their way by force are good, worthy of survival. Natural selection requires barbarianism; with mercy, pity, morality, socialism, and non-harm lacking. [28] Natural selection gives praise to those who kill and conquer.
Natural selection is a process, not an entity. From that same rebuttal of LaPointe:
quote:
There are a few things we need to deal with here. The first thing I would like to do is remind everyone to keep in mind that natural selection is a natural, mechanistic sorting process, not a conscious entity. . we should understand that we should no more expect mercy and pity (or hatred and brutality) from a process like natural selection, than we should from a process like radioactive decay - they are both processes, plain and simple, with no minds, desires, hopes, plans, etc. LaPointe unfortunately uses language that tends to personify natural selection as an evil, living entity that makes bloodthirsty demands from which we should run with revulsion and horror. Whether LaPointe does this purposely, or whether it is just an accidental slip on his part, is impossible to tell, but it is sufficient that we as readers make sure we are not misled by his anthropomorphizing language.
The next point I would like to make is that while natural selection causes organisms with higher fitness to predominate, fitness is not necessarily correlated with qualities such as brutality and selfishness, contrary to LaPointe's apparent beliefs. Natural selection definitely does not argue "that the best and fittest society would be one where its individuals look out only for themselves and would advance themselves, if possible, at the expense of others" (7-P1). It is not difficult to understand why this is the case. Imagine two populations, which we will creatively dub population A and population B. The people in A peacefully coexist with one another. The people in B, however, literally stab one another in the back every chance they get. In B, if a stronger person encounters a weaker person, the stronger person kills the weaker person. Now, which society will last longer? While A is pretty stable (at least as far as its constituents go), it won't be long before B has disintegrated enitrely, with almost everyone dead. Selection processes favor a society like A. In fact, what would happen if a group of people in B developed a mutant behavior pattern of cooperating with one another for mutual defense? In some cases, a cooperative band would be able to survive any attack by an individual in B, and hence, when all the killing is done, what we have left is a cooperative population just like A! This is, of course, a simplistic account - more complex dynamics can be found in the explorations into game theory made by biologists like John Maynard Smith and R.A. Fischer. Richard Dawkins gives a readable account of game theory in The Selfish Gene and River Out of Eden. But anyways, you get the point - selective processes operating on the biological world do not necessarily favor, much less "require" brutality and selfishness. Natural selection often favors the emergence of the social characteristics LaPointe claims disprove natural selection.
Point 55 refuted, once again thanks to Mark I. Vuletic.
Point 56:
Natural selection is also unplanned, making evolution of any part of our bodies or other organisms impossible, because evolution cannot form fingers starting from the wrist, if the whole process is unplanned. How would it know when to proceed and what to do next, if it is all unguided?
Why does evolution need to be planned? Is climate planned? Are orbits planned? You seem to believe that everything in nature is a consequence of some grand architectural drawing and divine micromanagement rather than the action of the laws of physics. Your argument here denies all of science in favor of some medieval serf-type mentality that requires the hand of god (I use lowercase when I believe the entity discussed is God being misinterpreted) to personally create tornados and hurricanes and push the planets along. This is not remotely a modern world understanding of how things work.
I also seriously doubt you behave in society in a manner consistent with your arguments. Do you really tell others god made it rain today, or I don’t need to work because god will provide, or terrorism is god’s will, so it is useless to counteract terrorism because doing so would be going against the will of god?
As to the fingers from the wrist, here is a picture of how that appears to have happened from:
Page not found | Skeptical Inquirer
Point 56 has some need of further explanation of why physics should be abandoned in favor of the ever present invisible hand of god of medieval times as the explanation of all change and motion IMHO.
Aside from natural selection, there are other problems with evolutionary thought.
Well, I’m here to answer such problems to the best of my ability. I think many of these so-called problems are due to misunderstanding what evolution means.
Edited by anglagard, : contains not creates antibodies
Edited by anglagard, : forgot the quote box in one area

This message is a reply to:
 Message 62 by anglagard, posted 10-29-2006 1:20 AM anglagard has not replied

Replies to this message:
 Message 67 by Someone who cares, posted 10-31-2006 12:14 AM anglagard has not replied
 Message 70 by Someone who cares, posted 11-04-2006 9:46 PM anglagard has not replied

  
anglagard
Member (Idle past 836 days)
Posts: 2339
From: Socorro, New Mexico USA
Joined: 03-18-2006


Message 64 of 100 (359741)
10-29-2006 10:03 PM


Biogenesis, Damned Lies, and Probability
Point 57:
The law of biogenesis states that life must come from life, but evolutionists say, that in the early history of the earth, life came from non-life. [29] What absurdity! Also, the cell principle states that all existing cells had to have come from earlier, already existing cells. So if the first cell didn’t come from a preexisting cell, that is going against the cell principle. How could the first cell have even come together? There is no outside guiding force in evolution, so what told the first cell to gather its parts together and function and continue evolving and getting more complex? What told the cells to combine into the various tissues we see in animals? Or what told the cells to differentiate into those different kinds of cells neccessary for the survival of the organism?
The study of how life originated is called abiogenesis. The study of how life changed once it exists is called evolution. These are two different areas of scientific inquiry. Therefore the concept of abiogenesis should properly be addressed in an essay titled “Abiogenesis” or “Evolution and Abiogenesis.”
However, since you are arguing for an overly simplistic view of abiogenesis, I will provide a picture to help. From Lies, Damned Lies, Statistics, and Probability of Abiogenesis Calculations
I have already addressed what told evolution to exist in the previous post, namely that it was not “told” but rather is a natural consequence of chemistry and physics.
Since point 57 is not actually about evolution, I consider it outside of this topic.
Point 58:
How about the chances for evolution to occur? Can we put any hopes in that? Let’s see: Professor Morowitz in his book Energy Flow in Biology estimates that the probability of even the most simple living organism forming by chance is one chance in about ten to the 340,000,000th power! [30] How about the formation of the genetic code? Any chance there? Well, let’s see. The probability for the genetic code being made by chance is . well, not even mathematically possible! How about DNA? Same story! Mathematically impossible! You can imagine how much faith it takes to believe in something that is mathematically impossible! Not even five million years will help! There would not be nearly enough time for a single cell or something even simpler, to form into a complete, fully developed human, even if we gave it that ridiculously low chance of forming the single cell!
Assigning probabilities of existence to something that already exists is ridiculous. From that same source Lies, Damned Lies, Statistics, and Probability of Abiogenesis Calculations
quote:
The very premise of creationists' probability calculations is incorrect in the first place as it aims at the wrong theory. Furthermore, this argument is often buttressed with statistical and biological fallacies.
At the moment, since we have no idea how probable life is, it's virtually impossible to assign any meaningful probabilities to any of the steps to life except the first two (monomers to polymers p=1.0, formation of catalytic polymers p=1.0). For the replicating polymers to hypercycle transition, the probability may well be 1.0 if Kauffman is right about catalytic closure and his phase transition models, but this requires real chemistry and more detailed modelling to confirm. For the hypercycle->protobiont transition, the probability here is dependent on theoretical concepts still being developed, and is unknown.
However, in the end life's feasibility depends on chemistry and biochemistry that we are still studying, not coin flipping.
Point 58 is meaningless.
Point 59:
Now let us observe this, if in the past four or so thousand years, no evolution of any organisms has occurred, all kinds of organisms have remained relatively unchanged, the only changes being slight variations within kinds, then what is the chance that it would happen in the time evolutionists give it?
Evidently the fuzzy term “relatively unchanged” allows for new species as per rebuttal to Point 51.
Point 59 refuted. New species in the last 4000 years means evolution has occurred.
Point 60:
By the way, the simple cell is not so simple! It is like a miniature factory that produces products, maintains itself, and works together with other cells to make tissues, organs, systems, and in the end result, a body. Take a plant cell, it should be simpler than an animal cell, right?
I would say wrong. Here is an animal cell:
Here is a plant cell:
Which one is simpler?
Point 60 refuted unless and until a realistic answer to the question of which is simpler? is provided.
Point 61:
Yet man still doesn’t fully understand how the chlorophyll converts solar energy into chemical energy!
They seem to have a pretty good idea here:
Light-dependent reactions - Wikipedia
Point 62:
Or, exactly how meiosis of the cell works.
Since you (not the properly educated man, or indeed woman) does not understand, perhaps a tutorial is in order. Try:
http://www.biology.arizona.edu/...utorials/meiosis/main.html
I’d say point 62 refuted depending upon the definition of “exactly.”
Also, the cell needs so many interrelated parts to function, like: proteins, DNA, RNA, nucleotide molecules, lipids, sugars, amino acids, cell membrane or wall, phosphorous, calcium, sodium, potassium and other elements. How could all of this have originated by chance? I think the answer is simple.
Therein lies the problem, the answer is there to some extent, but it appears it is not as simple, or indeed simplistic, as you demand.
Yet, there are still some more difficulties with evolution, aside from probabilities.
Haven’t seen an insurmountable difficulty yet that could not be explained by using one’s noggin (or if necessary, exercising one’s dormant critical thinking skills).
Edited by anglagard, : bad link

Replies to this message:
 Message 71 by Someone who cares, posted 11-04-2006 10:22 PM anglagard has not replied

  
anglagard
Member (Idle past 836 days)
Posts: 2339
From: Socorro, New Mexico USA
Joined: 03-18-2006


Message 68 of 100 (360017)
10-31-2006 12:54 AM
Reply to: Message 66 by Someone who cares
10-30-2006 11:53 PM


A Few Housekeeping Matters Before Proceeding
Point 46: You have a problem with the human spine and knee structure? I give you one week to propose something better to perform the same functions and to equal or surpass the design we have. Go on. That's a challenge.
As I said, I will focus on going through the entire essay and allow you to respond to each objection before I critique each response, otherwise we would still be debating the definition of transitional fossils.
If I say I am going to do something, then that is what I am going to do regardless of any "challenges." I would have liked to have responded to some of Holmes' and Iano's replies to me in another thread but I vowed, rightly or wrongly, to not post in that thread again. Therefore, regardless of taunts, abuse, or even regrets, I will keep my word once made.
I will respond to your challenge after I am done with a single response to each point all the way to the end of the essay, as best I can define them, as promised.
And again, calling someone names who has a different view than you doesn't show respect and good sport.
I did not promise I would not get carried away with my human frailty toward going too far on occasion, only that I would try to avoid it. Obviously you feel I did in this case and for any personal insult you may feel I apologize as I am not arguing against your person but rather the essay. However, that being said, I believe it is important for you to realize that some of your statements as represented in the essay and in the replies are simply outside the bounds of logic and critical thinking as I and many others who are very familiar with such concepts understand. I will try to be as gentle as I can muster in pointing out there are such flaws in the reasoning presented but I will not gloss over such errors simply to be accomodating to any personal identification you may make with the statements made in the essay.
I will continue in the same vein as I patiently have so far in my next post, so that the entire essay can be examined with a minimum of interruption. Further challenges made prior to getting to the end of the essay will be ignored from this end.
Please do not respond to this post if you feel the same way about getting to the end of the essay first, then taking on issues raised by the responses second.
Edited by anglagard, : clarity

This message is a reply to:
 Message 66 by Someone who cares, posted 10-30-2006 11:53 PM Someone who cares has replied

Replies to this message:
 Message 69 by Someone who cares, posted 11-04-2006 8:54 PM anglagard has not replied

  
anglagard
Member (Idle past 836 days)
Posts: 2339
From: Socorro, New Mexico USA
Joined: 03-18-2006


Message 72 of 100 (361927)
11-05-2006 4:03 PM


Abiogenesis is not Evolution
From where we let off:
The following portions of the essay are about abiogenesis, speculation on how life began, which is different from the Theory of Evolution, which is about how species changed over time once created. In my opinion, confusing the two terms is quite common among YECs and is also used as a propaganda technique to conflate the ToE, which has a huge amount of favorable evidence with abiogenesis, a field which at present contains less empirical evidence and more speculation. The other purpose of the propaganda technique is to attack all other creationists, such as Old-Earth, Hindu, and some other Theistic positions, in the hope of enlisting their conversion to the war against all science and history as revealed by God and Nature and to limit inquiry into the overly-simplistic methods used by YECs to interpret the Bible.
However, I will examine this portion of the essay to clarify any potential misconceptions concerning abiogenesis to limit the amount of false information and understanding of the term. I once again clarify that my argument is against the position as represented in the essay, not the person who wrote it. I seriously doubt that S1WC will change his fundamental position as a result of my criticism, but if nothing else, it may result in an improvement of the essay, as he has indicated.
Point 63:
A few problems exist with the early atmosphere of the earth, where the first cell parts, by common evolutionary thought, were to come together in a body of water. Such as the fact that amino acids are destroyed by oxygen. So, in order for these “building blocks of life” to survive, the early atmosphere of the earth would have to have been oxygen free. But then another problem arises, if there was no oxygen, there would be no ozone layer. No ozone layer means that ultraviolet rays of the sun would reach the earth.
The primary evidence that the Earth had minimal free oxygen in the early atmosphere is the presence of banded iron formations. From The atmosphere - origin and structure
quote:
Evidence from the Rock Record
” Iron (Fe) i s extremely reactive with oxygen. If we look at the oxidation state of Fe in the rock record, we can infer a great deal about atmospheric evolution.
” Archean - Find occurrence of minerals that only form in non-oxidizing environments in Archean sediments: Pyrite (Fools gold; FeS2), Uraninite (UO2). These minerals are easily dissolved out of rocks under present atmospheric conditions.
” Banded Iron Formation (BIF) - Deep water deposits in which layers of iron-rich minerals alternate with iron-poor layers, primarily chert. Iron minerals include iron oxide, iron carbonate, iron silicate, iron sulfide. BIF's are a major source of iron ore, b/c they contain magnetite (Fe3O4) which has a higher iron-to-oxygen ratio than hematite. These are common in rocks 2.0 - 2.8 B.y. old, but do not form today.
” Red beds (continental siliciclastic deposits) are never found in rocks older than 2.3 B. y., but are common during Phanerozoic time. Red beds are red because of the highly oxidized mineral hematite (Fe2O3), that probably forms secondarily by oxidation of other Fe minerals that have accumulated in the sediment.
Conclusion - amount of O2 in the atmosphere has increased with time.
Any free oxygen would have been largely consumed in the reactions that created the massive deposits of banded iron formations and {ABE- the allowed} red beds.
It is also important to note that amino acids have been found in meteorites, indicating such presence and uniqueness is not limited to an earthly environment. Additionally, asymmetry in the right and left handed forms of the amino acids, similar to that in life, is also present in such extraterrestrial bodies. See Amino Acid Asymmetry in the Murchison Meteorite! by Brig Klyce for more details. Also from Glen R. Morton see the beginning of http://home.entouch.net/dmd/wrong.htm where:
quote:
"Using sensitive analytical techniques, the researchers extracted and studied in meticulous detail four amino acids found in the carbon-rich Murchison meteorite, a type known as a carbonaceous chondrite. They examined amino acids that were relatively common in the meteorite but were not among the 20 amino acids found in terrestrial organisms.
"By focusing on these particular amino acids, Cronin and Pizzarello could avoid problems of contamination that would bias the results. In each case, the researchers found an excess of the left-handed form of the amino acid, ranging from 2 to 9 percent.
"The findings indicate that even amino acids that are never found in known life-forms, and so could not result from terrestrial biological evolution, display a lef-handed bias."~I. Peterson, "Left-handed Excess in Meteorite Molecules", Science News, Feb 22, 1997, p. 118
It is interesting, yet unmentioned in the essay, that the presence of banded iron formations are evidence for an old Earth and an atmosphere significantly different from that of today. But the arguments for an old Earth must await the next set of posts.
Point 64:
Then these ultraviolet rays of the sun would inevitably destroy any amino acids. Therefore making the whole idea of amino acids living and joining to form a cell impossible! To add on to the whole problem, if oxygen didn’t exist, then ultraviolet rays of the sun would break apart hydrogen molecules, releasing oxygen, which would, in effect, destroy the surrounding amino acids floating in the water!
Strange such amino acids exist in meteorites but are considered impossible on Earth. One would think the environment in space, without its ”protective ozone layer’ would not allow for amino acids to exist. Also, from http://hwiki.embl.de/wiki/index.php/Origin_of_life
quote:
For example, from spectral analyses, organic molecules are known to be present in comets and meteorites. In 2004, a team detected traces of polycyclic aromatic hydrocarbons (PAH's) in a nebula, the most complex molecule, to that date, found in space. The use of PAH's has also been proposed as a precursor to the RNA world in the PAH world hypothesis.
Additionally, how would ultraviolet light reach life two miles down in rock as shown to exist in the rebuttal to point #35?
Point 65:
Also, as a part of this idea of cell parts forming in the early atmosphere, scientists say that certain gases, such as methane, ammonia, and others, had to exist with no oxygen, to help out the formation of the first living cell. But, this is just an idea to help this theory work, it has no evidence that it actually was this way in the early atmosphere of the earth.
Are you denying the existence of this:
and this
These are banded iron formations that can only be created in an atmosphere without significant oxygen. Or are you arguing that chemistry, along with all bioscience and geoscience is also completely wrong?
Point 66:
Again, without oxygen, we see the destructive powers of ultraviolet rays of the sun, which would make ammonia powerless to do its job. Ultraviolet rays of the sun would turn methane gas into some heavier hydrocarbons. [31] Also making it unsuitable for the job. In addition, the atmosphere scientists used to make a few amino acids in labs is not the type of atmosphere which would be of the early earth.
Apparently the ”atmosphere’ of space is sufficient. So what is the atmosphere of the early Earth that fits all geologic, biologic, and chemical evidence? Also, any hydrocarbons would have helped create a favorable environment for abiogenesis under some models. Since volcanoes even today manage to continuously spew out the compounds that are supposed to make up the different early atmosphere, a source for such compounds is easily imagined. As seen from The atmosphere - origin and structure
quote:
Produced by volcanic out gassing.
” Gases produced were probably similar to those created by modern volcanoes (H2O, CO2, SO2, CO, S2, Cl2, N2, H2) and NH3 (ammonia) and CH4 (methane)
” No free O2 at this time (not found in volcanic gases).
” Ocean Formation - As the Earth cooled, H2O produced by out gassing could exist as liquid in the Early Archean, allowing oceans to form.
o Evidence - pillow basalts, deep marine seds in greenstone belts.
Point 67:
We have probably heard of the event when Claudia Huber and Gunter Wachterhauser supposedly "made life." But the real story is, they only made a few amino acids, some linked in small groups. This is not even nearly close to being a simple living cell! Also, chemical compounds composed of molecules tend to change and act up when in water, so how could the first cell parts have originated and combined in water?
The only people I can find in a Google search asserting that others stated they supposedly made life are YECs such as in:
However even AIG at Missing Link | Answers in Genesis states:
quote:
However, as shown above, all Huber and Wchterhuser produced were a few dipeptides and even fewer tripeptides. While they didn’t make the deceitful claim quoted above, their evolutionary faith means that they see far more significance in their experiment than it deserves.
Obviously the prospect of synthesizing life scares the hell out of YECs. No wonder it is so important for them to conflate abiogenesis with evolution, a strategy that will backfire when life is created in the laboratory or even possibly discovered on other bodies in the solar system.
Point 68:
Also worthy of mention is, that the amounts of carbon monoxide used in those experiments were much larger than actual amounts found in nature. [32] Plus the amounts of right and left handed molecules used in those experiments were not equal, as would be the real case with real water in the early environment. So those conditions were not like the ones that would be in the real atmosphere of the earth. Therefore, cell parts would not have been able to form and come together in the real early earth’s atmosphere.
One incomplete experiment does not render the entire concept of abiogenesis ”proven wrong forever.’ Each experiment provides more insight which leads to better results, which is the way science works. Huber and Wchterhuser are not the final word on all abiogenesis experiments that will ever be performed in the future.
There are competing theories on how life originated on Earth including
quote:
” 1 Current models
” 1.1 Origin of organic molecules
o 1.1.1 Miller's experiments
o 1.1.2 Eigen's hypothesis
o 1.1.3 Wchtershuser's hypothesis
” 1.2 From organic molecules to protocells
o 1.2.1 "Genes first" models: the RNA world
o 1.2.2 "Metabolism first" models: iron-sulfur world and others
o 1.2.3 Bubble Theory
o 1.2.4 Hybrid models
” 2 Other models
” 2.1 Autocatalysis
” 2.2 Clay theory
” 2.3 "Deep-hot biosphere" model of Gold
” 2.4 "Primitive" extraterrestrial life
” 2.5 The Lipid World
From Abiogenesis - Wikipedia. Please refer to the linked article for more details.
Also, obviously if the amount of chirality, that is right and left handed amino acids (not water), is already not equal in meteorites, then it is not to be expected ”in nature.’
There is much speculation in the field of abiogenesis. This should be an interesting contrast to the next paragraph of the essay which actually asserts geologic evidence exists for Noah’s Flood!
Edited by anglagard, : clarity

Replies to this message:
 Message 73 by Someone who cares, posted 11-08-2006 11:04 PM anglagard has not replied

  
anglagard
Member (Idle past 836 days)
Posts: 2339
From: Socorro, New Mexico USA
Joined: 03-18-2006


Message 75 of 100 (363824)
11-14-2006 9:18 PM


The YEC Stand Against All Geoscience
Point 69:
Now let’s examine a little different view. What do the fossil finds have to say about something like, a universal flood, such as Noah’s flood? For example, there are over 500 cases in the fossil finds, where “simpler” organisms were found in “more recent” layers, then some “more advanced” organisms. [33] Now evolution cannot account for this.
Geoscience easily accounts for this. Fossils may erode out of older strata and be deposited in newer strata, just like the cobbles erode out of older strata to make up newer strata in a conglomerate, or sand is weathered from rock by water or wind and is transported then recompacted as sandstone. That is how sedimentary rocks are made. Additionally there are 250 million cataloged fossils, is it any surprise that an insignificant 500 may have been moved from their original location (like all the materials that make up a conglomerate or sandstone type rock). Also, this 500 apparently includes all fossils discovered in overthrust belts, which is where the geologic sequence is turned upside down in a well understood manner. Ignorance of basic geologic processes in sedimentology means nothing to the overall truth or falsehood of evolution.
For further information see:
CC340: Out-of-place fossils
Point 70:
But in a universal flood, this could very well happen, with all the swirling water and sediments settling in various places at various depths.
Does all this swirling water which can supposedly erode granite and basalt to a 20 km depth known as continents within 40 days have any effect on a boat bobbing on the surface? Swirling water is not a bunch of giant jackhammers or a series of multi-million- megaton underground nuclear explosions. No wooden boat could survive such a physical process that would redistribute the Earth’s crust in 40 days. The heat generated in the physical force necessary to pulverize 20 km of rock to sediment alone would melt, vaporize, and boil off boat, crust, ocean, and atmosphere.
Additionally, a global flood once in settling mode would leave one type of global sediment. How does a global flood deposit shale here, sandstone there, limestone yet over there, delicate diatoms in chalk at yet another place, or indeed igneous and metamorphic rock, if that is your proposal? In your parlance, how does a global flood know when to deposit which type of rock at which time, particularly considering layers of different sediments at the same place, since it would create the same depositional environment worldwide?
Water also has limits in its ability to carry sediment up to the point it is “swirled” so fast it boils. The huge amount of proposed sediment in such water would make it Noah’s Mud, not Noah’s Flood.
Point 71:
But why would most “lower” species be under the “more advanced” ones? Simple. Since the “lower” species are typically smaller and less capable of swimming against large currents of water, they would be pulled down to settle lower and quicker in the dirt and sand at the bottom of the body of water. But the more “advanced” creatures are typically stronger and larger, and can swim against the currents and find temporary safe places, so they would settle later, and higher, since the sediments would be building up higher and higher with time during the flood.
According to the Earth’s acceleration due to gravity combined with resistance to settling from the relative density of water known as hydrodynamics, small less dense objects settle more slowly than large more dense objects in water. Therefore your “typically smaller” species do not settle “quicker in the dirt and sand at the bottom of the body of water.” For example, chalk consists of diatom fossils 700 to 1000 angstroms in diameter settle in this gravitational field at a rate of .0000154 mm/sec in still water. In one year, such as the proposed flood, they would have created a maximum around half a meter of chalk. Guess under flood conditions one well known chalk formation should be referred to as the white speed bumps of Dover.
I also question the swimming ability of various life forms such as grass and flowering plants which are not found in strata lower than the Cretaceous. I suppose such organisms also fled uphill as the flood water rose.
Point 72:
Humans, they could maybe reach higher ground if they saw the flood coming, and when the flood would come, they could grab on to logs and float for awhile, until they gave in and couldn’t hold on much longer, or some could swim searching for higher ground, and sink when they wouldn’t have the strength to keep above the waters. Thus they would settle later than most animals, and they would be higher in the sediments. Natural settling action and fighting for life, not evolution!
Humans would have settled faster, not slower than tiny organisms as shown in point 71 and therefore should be lower in the fossil record than diatoms. Additionally any sorting in the fossil record according to any global flood would have to explain the following observations:
quote:
1. the extremely good sorting observed. Why didn't at least one dinosaur make it to the high ground with the elephants?
2. the relative positions of plants and other non-motile life. (Yun, 1989, describes beautifully preserved algae from Late Precambrian sediments. Why don't any modern-looking plants appear that low in the geological column?)
3. why some groups of organisms, such as mollusks, are found in many geologic strata.
4. why organisms (such as brachiopods) which are very similar hydrodynamically (all nearly the same size, shape, and weight) are still perfectly sorted.
5. why extinct animals which lived in the same niches as present animals didn't survive as well. Why did no pterodons make it to high ground?
6. how coral reefs hundreds of feet thick and miles long were preserved intact with other fossils below them.
7. why small organisms dominate the lower strata, whereas fluid mechanics says they would sink slower and thus end up in upper strata.
8. why artifacts such as footprints and burrows are also sorted. [Crimes & Droser, 1992]
9. why no human artifacts are found except in the very uppermost strata. If, at the time of the Flood, the earth was overpopulated by people with technology for shipbuilding, why were none of their tools or buildings mixed with trilobite or dinosaur fossils?
10. why different parts of the same organisms are sorted together. Pollen and spores are found in association with the trunks, leaves, branches, and roots produced by the same plants [Stewart, 1983].
11. why ecological information is consistent within but not between layers. Fossil pollen is one of the more important indicators of different levels of strata. Each plant has different and distinct pollen, and, by telling which plants produced the fossil pollen, it is easy to see what the climate was like in different strata. Was the pollen hydraulically sorted by the flood water so that the climatic evidence is different for each layer?
From Problems with a Global Flood, 2nd edition
I personally have fossils of ammonites, trilobites, and enough crinoid stems to fill a 10 pound sack, all of which were found on the surface. Do you deny their existence along with the coins of Valens and Diocletian, within my touch?
Point 73:
Also, in many places of the earth, we have found trees and animals that were fossilized upright, and going through several geologic layers. So does this mean they were “millions and millions” of years old?!? Of course not! But, if there was a great flood, this would naturally happen! The great flood waters would rip trees out of the ground, and then the trees would get stuck in the mud in any position, even upright.
These are known as “polystrate” tree fossils. As can be seen at http://www.talkorigins.org/faqs/polystrate/trees.html such fossils, which usually include root systems too delicate to survive a global scouring, have been observed and explained for well over 100 years as a result of local flooding and subsequent deposition (not erosion). The majority of such tree fossils have been buried by local mudslides and floods as shown by the following:
quote:
Like the modern environments around Mt. St. Helens, there is potential to bury stumps in-place *and* to transport them upright in a variety of sedimentary environments (although burial in-place is far more common). Distinguishing the two (or even recognizing the presence of both) is not difficult. To simply say, "tree stumps can be transported, so all occurrences can be dismissed", is incorrect. The vast majority of occurrences can not be explained by transport.
Source: http://www.talkorigins.org/faqs/polystrate/yellowstone.html
The existence of multiple layers of such polystrate fossils also begs the question which layers were localized floods and which layer was the global flood.
From http://www.geocities.com/earthhistory/forests.htm
quote:
The upright trees at Specimen Ridge are rooted in fine-grained tuffaceous sandstone and encased in conglomeratic mudflows. The grain size of the conglomerate decreases away from the location of the volcanic source areas, the East and West Absaroka belts. Also, the ratio of upright, in situ trees to horizontal trees increases away from the Eocene volcanic source areas - the eruptions and mud flows flattened whole forests proximal to the source, while many trees are preserved in growth position in more distal locations, such as at Specimen Ridge. Although it is unclear how many successive forest layers are present in the Lamar River Formation, estimates range from 9-12 for Specimen Ridge. Some of the levels have very wide and old trees trunks.
Point 74:
Same with animals. A universal flood explains it, but evolution can’t! There is much evidence that the geologic layers formed suddenly, and that is what a universal flood would do, form many layers of sediments quickly, one on top of the other.
Absolutely False! There is not a single piece of actual scientific evidence that the Earth’s geologic layers formed suddenly. If there is even one then name it.
However, here are 29+ categories of millions of pieces of evidence that the Earth’s geologic record did not form suddenly:
Radiometric dating - All common forms of radiometric dating, including C14, K-Ar, Ar-Ar, Rb-Sr, Th-Pb, U-Pb, and fission track. The dates derived from these diverse methods, when properly interpreted rather than intentionally misapplied, show that all but the very most recent deposits in the geologic column is vastly older than any postulated flood. The argument that all radioisotope data is false, regardless of correlations to other independent methods of verification, is argued in more detail in a subsequent paragraph in the essay. Further examination and rebuttal of this claim will appear there.
Paleomagnetism - Because the Earth’s magnetic field has reversed polarity and has wandered over the globe in the past, certain igneous rocks show such preferred magnetic orientations when sufficiently cooled. By mapping these directions and reversals, which correlate with radioisotope dating and stratigraphy, it is easily shown that the vast majority of seafloor sediments, along with most volcanic rock, are way too old to have been deposited by any flood. In fact such measurements are one of the great evidences for plate tectonics, which alone invalidate a global flood.
Water in confined aquifers - The chemistry of water may be measured as to its constituents, as any municipal water authority already knows. Under flood conditions, the water chemistry in a confined aquifer would have changed as lower elevation aquifers would contain more salt than higher aquifers as the flood waters diluted the salt content in the recharge zone. Not only is this behavior not shown by any known confined aquifer, but the age of such water according to the laws of physics is vastly older than any flood may have deposited.
Lake sediments aka “varves” - How does one create 20 million annual layers, each layer which would have taken at least a month to settle due to hydrodynamics as is observed in the Green River Formation?
Angular unconformities - Angular unconformities are where sediments are laid down in layers, then tilted and eroded, then new sediments are deposited on top. How does a global flood simultaneously deposit, tilt, and erode in the same exact place?
Sedimentation rates - Why would there be Precambrian rocks below ones feet in the Canadian Shield area, yet the entire geologic column in the Williston Basin in North Dakota? Why would a global flood scour down to the Precambrian in one place yet at the same time deposit tens of thousands of feet of sediment in another when it is exactly the same process? Giant post-pyramid ice ages are not an explanation as there is no written record or other evidence of increased historical glaciation to the extent needed to scour the Canadian Shield down in the last 4500 years, not to mention such Precambrian rocks elsewhere on Earth like South Africa.
Volcanism - According to ”flood geology’ every igneous rock layer that overlays sedimentary rock would have to be less than 4500 years old. Yet, historical records indicate this tremendous amount of simultaneous volcanic activity could not have occurred in recent times because someone would have noticed, becoming extinct and all when the atmosphere becomes unbreathable. Such a position directly contradicts the existence of the Deccan Traps which are up to 2 km thick and 500,000 square km in extent, yet supposedly erupted in India despite any historic evidence, after such a flood.
Ore deposit formation rates - Most ore deposits require a longer period of time to separate their constituent elements and then cool to create an economically viable source of minerals. From iMIS :
quote:
In the intrusive, diagenetic, and metamorphic setting, the total duration of the geologic events associated with mineralization is tens of millions of years, but in all three settings there is ample evidence that episodic fluid venting and ore formation are short-lived, on the scale of tens to hundreds of thousands of years or less. Remarkably, the very different geologic settings covered in this volume provide quite a consistent story, undoubtedly for very different reasons. In contrast to the perception that motivated the symposium, the papers in this volume find no discrepancy between estimates of the duration of hydrothermal activity and/or ore deposition derived from modeling and derived from radiometric dating. Indeed, both approaches indicate pulses of fluid expulsion (magmatic, hydrocarbon, or aqueous) in a much longer-lived framework of intrusions and volcanism, sedimentation, or continental scale mountain building and erosion. The old view that it takes many, many millions of years to create an ore deposit is correct in the broadest sense, but we now can appreciate the brevity of the actual ore-forming episodes themselves.
Banded iron formations and red beds - As covered under point 63, banded iron formations can’t form in the presence of significant oxygen. Yet they were supposedly deposited in a flood which supposedly allowed animals to breathe both before and after.
Tree ring data - Rings on currently living trees that indicate they are older than 4500 years do not indicate that they were drowned and died at the time of any proposed global flood.
Ice core data - Ice cores should show an annual layer consistent with a global flood with different salinity and oxygen isotope ratios, provided they were not destroyed by such a flood. In fact, such a flood would have melted the ice caps, which would not have been able to redevelop due to such minimal precipitation within a mere 4500 years.
Ocean core data - Ocean cores would show unsorted piles of terrestrial life and different distributions in grain sizes than observed. They would also show little difference in thickness between the mid Atlantic ridge and near subduction zones, which is not observed.
Inconsistent worldwide geologic formations - Any flood would have left a single layer of similar sediment worldwide, not the tremendous amount and variety of layers that exist in each of the thousands of boreholes and outcrops in geology.
Worldwide iridium layers - Although any worldwide flood evidence is lacking, there is a worldwide iridium layer at the K-T boundary where it exists. How could this iridium layer have been deposited among all those swirling waters in a flood?
Fossil record sorting - See response point 72
Formation of geologic features such as mountains and valleys -
How did something like the Himalaya Mountains form without anyone noticing all those earthquakes? How were valleys cut between such mountains in less than 4500 years?
Batholith formation - Why isn’t the Sierra Nevada granitic batholith still hot as it would have taken 90 million years to cool?
Detailed layering -
From Problems with a Global Flood, 2nd edition
quote:
One formation in New Jersey is six kilometers thick. If we grant 400 days for this to settle, and ignore possible compaction since the Flood, we still have 15 meters of sediment settling per day. And yet despite this, the chemical properties of the rock are neatly layered, with great changes (e.g.) in percent carbonate occurring within a few centimeters in the vertical direction. How does such a neat sorting process occur in the violent context of a universal flood dropping 15 meters of sediment per day? How can you explain a thin layer of high carbonate sediment being deposited over an area of ten thousand square kilometers for some thirty minutes, followed by thirty minutes of low carbonate deposition, etc.? [Zimmer, 1992]
Fossil forests - How could several layers of forests be found in the fossil record as per response to point 73? Which one is a global and which ones are local, and why can’t one be distinguished from the other?
Heat of formation -
Once again from Problems with a Global Flood, 2nd edition
quote:
If the geologic record was deposited in a year, then the events it records must also have occurred within a year. Some of these events release significant amounts of heat.
” Magma. The geologic record includes roughly 8 x 10E24 grams of lava flows and igneous intrusions. Assuming (conservatively) a specific heat of 0.15, this magma would release 5.4 x 10E27 joules while cooling 1100 degrees C. In addition, the heat of crystallization as the magma solidifies would release a great deal more heat.
” Limestone formation. There are roughly 5 x 10E23 grams of limestone in the earth's sediments [Poldervaart, 1955], and the formation of calcite releases about 11,290 joules/gram [Weast, 1974, p. D63]. If only 10% of the limestone were formed during the Flood, the 5.6 x 10E26 joules of heat released would be enough to boil the flood waters.
” Meteorite impacts. Erosion and crustal movements have erased an unknown number of impact craters on earth, but Creationists Whitcomb and DeYoung suggest that cratering to the extent seen on the Moon and Mercury occurred on earth during the year of Noah's Flood. The heat from just one of the largest lunar impacts released an estimated 3 x 10E26 joules; the same sized object falling to earth would release even more energy. [Fezer, pp. 45-46]
” Other. Other possibly significant heat sources are radioactive decay (some Creationists claim that radioactive decay rates were much higher during the Flood to account for consistently old radiometric dates); biological decay (think of the heat released in compost piles); and compression of sediments.
5.6 x 1026 joules is enough to heat the oceans to boiling. 3.7 x 10E27 joules will vaporize them completely. Since steam and air have a lower heat capacity than water, the steam released will quickly raise the temperature of the atmosphere over 1000 C. At these temperatures, much of the atmosphere would boil off the Earth.
Aside from losing its atmosphere, Earth can only get rid of heat by radiating it to space, and it can't radiate significantly more heat than it gets from the sun unless it is a great deal hotter than it is now. (It is very nearly at thermal equilibrium now.) If there weren't many millions of years to radiate the heat from the above processes, the earth would still be unlivably hot.
Chalk formation - See rebuttal point 71.
Salt (evaporite) formation - The geologic record contains many examples of evaporite deposits including salt which are incompatible with a global flood.
From http://www.csun.edu/%7Evcgeo005/wise.htm
quote:
Thick salt beds formed by evaporation of sea water are a common feature of geologic columns in many parts of the world. The "young earth geologists" interpret almost all classic stratigraphic units as deposits produced during the flood year: hence, they must also account for interbedded salt formations as part of those events (Figure 6). Some of the more extensive salt formations with the U.S. are in the Jurassic of the Gulf Coast (Worrall and Snelson, 1989), the Silurian of the New York to Chicago region (Alling and Briggs, 1961; Smosna and Patchen, 1978), and the Permian of the Paradox Basin of Utah (Baars and Stevenson, 1982). In the center of the Paradox Basin these salts reach a depositional thickness of 1.5 km (Rocky Mountain Association of Geologists Atlas, 1972) with at least 29 separate cycles of salt deposition (Hite, 1960). To deposit just these beds in a single year would require the salt to form at an average rate of 4 meters per day (17 cm per hour or 2.8 mm per minute) - and this by evaporation during a world-wide flood event!
Deformed structures in metamorphosed sediments - There are areas in metamorphic rock where pebbles and even fossils have been stretched and deformed in processes that would have taken several times any 4500 years since a global flood.
Differential mineralization of fossils - How does one explain -
quote:
the replacement of the original material with a different mineral?
* Buried skeletal remains of modern fauna are negligibly
mineralized, including some that biblical archaeology says are
quite old - a substantial fraction of the age of the earth in this
diluvian geology. For example, remains of Egyptian commoners
buried near the time of Moses aren't extensively mineralized.
* Buried skeletal remains of extinct mammalian fauna show quite
variable mineralization.
* Dinosaur remains are often extensively mineralized.
* Trilobite remains are usually mineralized - and in different
sites, fossils of the same species are composed of different
materials.
How are these observations explained by a sorted deposition of
remains in a single episode of global flooding?
[From: jjh00@outs.ccc.amdahl.com (Joel J. Hanes)]
From The Skeptic Files - SkepticFiles Setting
Coal reef clocks - Coral reefs leave annual growth marks just like tree rings, they also leave daily growth markings. Older corals have shown by such daily evidence the Earth rotated faster than present, such as around 400 days per year in the Devonian. Essentially all coral shows this regardless of location, which matches other dating methods. The gradual decrease in the Earth’s rotation predicted by physics, along with the existence of corals older than the flood not predicted by flood geology indicates there was no flood.
Compression of all fossil life into too short a time period - If all species represented by fossils, coal, and petroleum from throughout the geologic record lived simultaneously, they would have been standing on each other, an ecological impossibility.
Too much organic material in fossil record -
Again from Problems with a Global Flood, 2nd edition
quote:
There are 1.16 x 10E13 metric tons of coal reserves, and at least 100 times that much unrecoverable organic matter in sediments. A typical forest, even if it covered the entire earth, would supply only 1.9 x 10E13 metric tons. [Ricklefs, 1993, p. 149]
Types of fossils inconsistent with flood - The vast majority of fossils are from shallow sea deposits, where are the intermixed terrestrial fossils one would expect from a global flood?
Relative erosion - Why are the Appalachian Mountains obviously more eroded than the Sierra Nevada if they were created simultaneously?
Surface features buried throughout geologic column -
Examples include:
1. rain drops;
2. river channels;
3. wind-blown dunes
4. beaches;
5. glacial deposits
6. burrows;
7. in-place trees
8. soils
9. dessication cracks;
10. footprints. [Gore, 1993, has a photograph (p. 16-17) showing
dinosaur footprints in one layer with water ripples in layers
above and below it. Gilette & Lockley, 1989, have several more
examples, including dinosaur footprints on top of a coal seam (p.
361-366).]
Thanks to Problems with a Global Flood, 2nd edition, The Skeptic Files - SkepticFiles Setting, http://home.entouch.net/dmd/dmd.htm#geo, and all those earth science and engineering professors at New Mexico Tech for much of this listing and its contents.
The above is limited to geological evidence. Scores of categories of more millions of examples of overwhelming evidence against a global flood also comes from physics, engineering, bioscience, linguistics, and history See the above links for more details.
As can be seen virtually the entire content and essentially all evidence from the geosciences directly refute a global flood. There are even more categories of evidence, should the above proves insufficient, but this reply was getting too long anyway.
It is also important to note that these independent dating methods correlate.
See Message 1 for what IMO is the best thread on EvC, especially considering it has never been refuted.
More on that when we hit the radioisotope dating paragraph.
Point 75:
Another interesting observation, in many places of the earth, we find groups of fossils; meaning that the animals were fossilized in groups. Now tell me, can evolution explain this? Why would animals pile together, die, and get fossilized one on top of the other?
The fact that there are layers of fossils does not mean they were simultaneously fossilized, just as layers of different kinds of rock does not mean the sediments were simultaneously deposited. Anaerobic conditions, such as at ocean, lake and marsh bottoms; tar pits; local catastrophes, such as volcanic ash falls, flash floods, mud flows, and avalanches; even differential transport, all set up specific areas where fossils are preserved, hence fossils are often discovered together. Most areas on land normally do not permit fossilization to occur. So what one is seeing is fossils are found in abundance where the conditions were right for fossils to occur, not that they all died everywhere at the same time under exactly the same conditions. Under a single great flood scenario, it would be more, not less, likely to find fossils everywhere.
Point 76:
In nature, the opposite happens, when animals die slowly, they usually go off alone to die somewhere away from the rest of the animals.
Which it appears you are insisting happened in every single case of fossilization. I am sure we are all quite familiar with conditions where the animals such as tunicates, corals, clams, and barnacles magically became unanchored to their surroundings and rushed off to die alone before all that volcanic ash covered them. Are you saying just like all those mammoths and other Pleistocene animals rushed off to die alone at La Brea, or humans at Pompeii and Herculaneum when Vesuvius erupted in 79 AD. They sure all ran off to die alone. Yup, just like Pelee in 1902 or even Sumatra last year, all those people went off to die alone.
How ridiculous.
Also, isn’t the idea that everything runs off to die alone a contradiction of the idea that everything fossilized simultaneously in a glop.
Point 77:
But say there was a universal flood? Imagine this, a flock of sheep or some other animals are peacefully grazing, and then they look up and see a huge wave coming at them. What do they do? They would most likely huddle together for protection and to shield their young. Then the waves would hit, all the dirt and trees and other matter would cover the sheep, and they would get stuck in mud, with most of them close by each other, as they were before the waves hit. So formed these groups of fossilized animals in piles.
Do you have any evidence of fossilized piles of domesticated animals living in harmony with all those fossilized piles of dinosaurs? The Flintstones is a cartoon, not a science treatise.
Point 78:
Here is the big proof though, fossils form when there is quick, tremendous pressure. Sound like a universal flood? Yes. Because without the pressure, the creatures would not fossilize. As an experiment, find some semi-hard mud and take an object, such as a tennis ball. Then, to represent evolution, set the object in the mud, toss some sand on it, and blow on it with a fan. Now pick up your object, did it leave a noticeable imprint? Probably not. Now, to represent the force of a universal flood, set the object in the mud, and get out your hose. Set the nozzle to full pressure, and turn it on. Direct the spray at your object. Now, pick up your object, did it leave an imprint? Most likely. See? Great force was needed. But it you have a local flood or just wind and dust, that would not have the pressure to leave an imprint in rock.
Big proof! More like big example of complete ignorance of the fossilization process. Here is three major ways that fossils form, permineralization, replacement, and compression. From Fossil - Wikipedia :
Permineralization -
quote:
Permineralization occurs after burial, as the empty spaces within an organism (spaces filled with liquid or gas during life) become filled with mineral-rich groundwater and the minerals precipitate from the groundwater, thus occupying the empty spaces. This process can occur in very small spaces, such as within the cell wall of a plant cell.
Replacement -
quote:
In some cases the original remains of the organism have been completely dissolved or otherwise destroyed. When all that is left is an organism-shaped hole in the rock, it is called a mould fossil or typolite. If this hole is later filled with other minerals, it is called a cast fossil and is considered a replacement fossil since the original materials have been completely replaced by new, unrelated ones. In some cases replacement occurs so gradually and at such fine scales that no "hole" in the rock can ever be discerned and microstructural features are preserved despite the total loss of original material.
Compression -
quote:
Compression fossils such as those of fossil ferns are the result of chemical reduction of the complex organic molecules composing the organism's tissues. In this case the fossil consists of original material, albeit in a geochemically altered state. This chemical change is an expression of diagenesis.
So how come you can use a garden hose to “prove” there is no such thing as permineralization but refuse to use a glass of water to prove larger sediments settle more quickly than extremely fine-grained sediments (like diatoms)?
Point 79:
Another point, if dead animals were just lying around, waiting to be covered by dust, then they would most likely be eaten, or decomposed by bacteria, and they would rot away; long before being covered. Then we wouldn’t get our fossil fuels! But in a universal flood, fossil fuels would be possible, and we have fossils fuels . so I think the evidence is clear.
Fossils are almost entirely made of mineralized bone or replaced by minerals. One reason is that all the other parts were eaten and decomposed by bacteria. I am unfamiliar with any bone-eating predators in the fossil record, please provide examples. As to petroleum deposits the fact that anaerobic conditions exist show how they were made. The two theories of formation require an old earth and no great flood as shown here:
Petroleum - Wikipedia
quote:
Biogenic theory
Most geologists view crude oil and natural gas, as the product of compression and heating of ancient organic materials over geological time. According to this theory, oil is formed from the preserved remains of prehistoric zooplankton and algae which have been settled to the sea bottom in large quantities under anoxic conditions. (Terrestrial plants tend to form coal) Over geological time this organic matter, mixed with mud, is buried under heavy layers of sediment. The resulting high levels of heat and pressure cause the remains to metamorphose, first into a waxy material known as kerogen which is found in various oil shales around the world, and then with more heat into liquid and gaseous hydrocarbons in a process known as catagenesis. Because most hydrocarbons are lighter than rock or water, these sometimes migrate upward through adjacent rock layers until they become trapped beneath impermeable rocks, within porous rocks called reservoirs. Concentration of hydrocarbons in a trap forms an oil field, from which the liquid can be extracted by drilling and pumping.
Abiogenic theory
The idea of abiogenic petroleum origin was championed in the Western world by astronomer Thomas Gold based on thoughts from Russia, mainly on studies of Nikolai Kudryavtsev. The idea proposes that hydrocarbons of purely geological origin exist in the planet. Hydrocarbons are less dense than aqueous pore fluids, and are proposed to migrate upward through deep fracture networks. Thermophilic, rock-dwelling microbial life-forms are proposed to be in part responsible for the biomarkers found in petroleum.
However, this theory is a minority opinion, especially amongst geologists and no oil companies are currently known to explore for oil based on this theory.
How does a great flood explain the existence of petroleum? People in the industry are betting millions of dollars a day on modern geology, yet I have never heard of one cent being spent to sink wells based upon “flood geology.” Why is that?
Point 80:
Plus, most fossils have been found to have water markings on them, and in a universal flood, water is plenty! Fossils would form by the minute!
Yes, in the oceans, which make up of the Earth’s surface water is plenty. In lakes, streams, or areas subject to periodic heavy flooding, water is plenty. It has even been assumed that it actually has rained in the past as evidenced by all the raindrop and hailstone casts throughout the fossil record. Where are the watermarks on the structures built by Egyptian and other civilizations? Where are such global flood-type watermarks in the wind-deposited sandstones that occur worldwide? Where are global flood watermarks in all those interbedded evaporate deposits, that according to flood geology should not exist in the first place?
Point 81:
We have a similar situation with coal. Coal, in the U.S., consists of tree bark in mud. Now calculate how much tree bark in mud you would find if a universal flood came by and ripped trees out of the ground, and they would become stuck in mud.
Coal is not just “tree bark in mud.” From Page not found | The Franklin Institute :
quote:
The first stage in coal formation is material composed of about 75 to 90 percent water plus twigs, leaves, branches, and other plant debris. Although peat itself is not coal, it is an important fuel used in Ireland and the Soviet Union.
Lignite
The second stage of coal formation is brown coal composed of compressed woody matter that has lost most of its moisture. It is used for local fuels in homes and industry. Germany uses its lignite to provide synthetic petroleum.
Bituminous
The third stage of coal formation is a dense, dark, brittle material that has lost all its moisture and most other impurities. It is ignited easily by a flame. Although bituminous coal is an efficient heating material, it produces a smoky yellow flame, ash, and sulfur compounds when it is burned. Strict emission laws have limited the amount of pollutants industries can release when this coal is burned. Bituminous coal is mined throughout the United States with major fields in the Appalachians, the Great Plains, and the Colorado Plateau.
Anthracite
Anthracite, sometimes called "hard coal," is the final stage in coal formation. Lignite coal and bituminous coal are sedimentary rocks. Anthracite is a metamorphic rock. It is found only in areas of mountain building where heat and pressure were great. Anthracite is the cleanest of all coals with the least impurities because it is mostly carbon. It does not produce as much heat as bituminous coal, but it is preferred because it burns cleaner and longer. Anthracite fields occur in northeastern Pennsylvania, Great Britain, and parts of the Soviet Union.
Despite any “flood geology” claims to the contrary, coal, particularly the metamorphic variety anthracite, takes longer than a post-flood timescale to form.
Point 82:
Another observation, the majority of our planet’s surface doesn’t even have the geologic layers in the order that scientists gave them to support evolution.
Absolutely False Name one place on Earth where the geologic layers are out of order according to the ideal geologic column, which is not easily explained by an overthrust or a discontinuity.
Point 83:
Also, Roger Lewin, a paleontologist, acknowledges that the mitochondrial DNA method is in support for Noah’s Ark story. [34]
NO, NO, NO - This is the whole paragraph directly from page 131 of the book:
quote:
These and other data lead Wilson and his colleagues to conclude that “the common ancestor of modern humans lived in Africa, about 200,000 years ago.” Moreover, they suggest, “when individuals from this population moved out of Africa into Europe and Asia, they did so with little or no mixing with existing local populations of more primitive humans.” In other words, the mitochondrial DNA technique appears to support the argument that modern humans evolved in one place and then migrated, replacing premodern populations - the Noah’s Ark hypothesis.
Obviously the “Noah’s Ark hypothesis” refers to modern humans evolving in one place then replacing the more primitive existing human population a few hundred thousand years ago, not that there was some global flood 4500 years ago.
From page 127 of the same book:
quote:
The Noah’s Ark interpretation (center) holds that modern humans evolved in Africa, then migrated throughout the world, completely replacing the Neandertals and other existing primitive human populations.
From page 128:
quote:
The second interpretation, which Howells labels the “Noah’s Ark hypothesis,” envisages a single geographical origin of Homo Sapiens Sapiens, in Africa. Populations of this group then migrated throughout the world, replacing the existing primitive human populations - including Neandertals - they encountered. In this model, anatomically modern humans radiated from the center of origin, like ripples on a pond, and racial roots could be quite recent and therefore shallow.
If you absolutely could not go to the index to see how the term was used in the immediately preceding pages, you could have easily done some research to independently determine what this term meant:
From http://uts.cc.utexas.edu/~bramblet/ant301/fourteen.html :
quote:
Theories that propose a recent single origin for modern populations and subsequent rapid replacement as they migrate into other areas have been called the "Noah's Ark" hypothesis (Wolpoff, 1989). The "out of Africa" or recent African evolution (RAE) model (Stringer and Andrews, 1988) is one of these. It proposes a sharp break between archaic and modern H. sapiens. In this model, early modern humans originated outside of Europe, probably in Africa, then migrated into Asia and Europe, where they replaced Neanderthals and any other archaic human populations (Bruer and Rimbach, 1990). In the RAE model, evolution of modern humans was a speciation event. It further proposes that more archaic human populations would not be fertile with modern humans so there are no expectations of regional admixture between modern and archaic forms. RAE makes these predictions:
1. Modern anatomical forms will be found earlier in Africa than other regions.
2. There will be no hybrids or intermediate forms during dispersal due to hybridization.
3. Modern anatomical form existed in Africa by 100,000 YBP.
This source you directly quoted, In the Age of Mankind by Roger Lewin, is a pop-science book from the Smithsonian about human evolution according to science, not YECism. To so blatantly quote mine it, when it is so easily checked, brings up questions in my mind concerning your cognitive ability and/or your integrity.
Point 84:
It is interesting to note as well, that in almost every nation and tribe on this earth we find the occurrence of a universal flood in their history. [35]
Humans were too busy building pyramids in Egypt, creating the Akkadian Empire under Sargon in Mesopotamia, writing down events that would later be chronicled in the Rig Veda in the Indus valley, or working bronze in China in the Xia dynasty to notice any worldwide flood, which they did not record at the time such a flood was to have occurred, and which appears to have had absolutely no effect upon their respective civilizations. This is particularly remarkable considering that the up to 2 km deep and 500,000 square km in extent Deccan Traps, which are underlain by sediment and therefore must be post-flood, appear to also have had no notice or effect on such civilizations, not even in the nearby Indus Valley. Such flood myths also differ considerably in the details, including dates and extent, indicating there was no single global flood.
It is important to note that nearly all cultures which have myths concerning a flood were also polytheistic. Is the commonality of polytheism an argument for its validity?
Edited by anglagard, : Clarify scientific notation (ex. using 10E4 instead of 10,000) and more clarity and a few minor spelling errors.

Replies to this message:
 Message 76 by Someone who cares, posted 11-15-2006 11:44 PM anglagard has replied
 Message 81 by Someone who cares, posted 11-30-2006 9:30 PM anglagard has not replied
 Message 82 by Someone who cares, posted 12-01-2006 9:49 PM anglagard has not replied
 Message 85 by Someone who cares, posted 12-14-2006 12:24 AM anglagard has not replied
 Message 86 by Someone who cares, posted 12-26-2006 8:06 PM anglagard has not replied

  
anglagard
Member (Idle past 836 days)
Posts: 2339
From: Socorro, New Mexico USA
Joined: 03-18-2006


Message 77 of 100 (364280)
11-17-2006 4:20 AM
Reply to: Message 76 by Someone who cares
11-15-2006 11:44 PM


Correction
Under points 70 and 71, I falsely ascribed chalk formation to diatoms when actually they are formed by a single celled algae called coccolithophores.
Thanks for the correction goes to the creator of this thread, Lithodid-Man. See Message 38 for more details.
Unfortunately for your argument, this means that such chalk deposits would have taken even longer to form as the coccolithophores only deposit chalk sediment at a rate of less than 1 millimeter per year.

This message is a reply to:
 Message 76 by Someone who cares, posted 11-15-2006 11:44 PM Someone who cares has not replied

  
anglagard
Member (Idle past 836 days)
Posts: 2339
From: Socorro, New Mexico USA
Joined: 03-18-2006


Message 78 of 100 (364581)
11-18-2006 4:24 PM


Vestigial Structures
Point 85:
Now for another look at evolution. Many evolutionists claim that the human body has vestigial organs which were “leftovers” from our supposed previous forms, on the pathway from a single cell to our state now. This has some difficulties, and we will now examine some. It was once believed that humans had 180 vestigial organs. Now, with better knowledge and research techniques, there are only a few listed vestigial organs, with even those being debatable. See, the further we progress in knowledge, the more we figure out that those “vestigial” organs actually have jobs and functions, some that you could not live without!
Vestigial structures indicate common ancestry as opposed to special creation since their presence under circumstances other than evolution would conclude an incompetent, or intentionally deceitful, designer. From Wikipedia at Vestigiality - Wikipedia :
quote:
Vestigial structures are often homologous to structures that are functioning normally in other species. Therefore, vestigial structures can be considered evidence for evolution, the process by which beneficial heritable traits arise in population over an extended period of time. The existence of vestigial organs can be attributed to changes in the environment and behavior patterns of the organism in question. As the function of the structure is no longer beneficial for survival, the likelihood that future offspring will inherit the "normal" form of the structure decreases.
The vestigial versions of the structure can be compared to the original version of the structure in other species in order to determine the homology of a vestigial structure. Homologous structures indicate common ancestry with those organisms that have a functional version of the structure.[8]

Sure, some scientists made mistakes in the past as Robert Wiedersheim did by listing 180 vestigial organs. However, unlike the way “creation science” usually operates, once the mistake is determined, it is corrected, as opposed to continuing to re-quote the same mistake forever. Vestigial organs are numerous throughout life, so in the interest of brevity, and to go beyond limiting such structures solely to humans, why not list the top 10 according to Live Science at Top 10 Useless Limbs (and Other Vestigial Organs) | Live Science :
1. Human appendix - to be covered in point 86
2. Male breast tissue and nipples - Obviously as unnecessary as teats on a Boar Hog, but clearly a leftover from the fact they are developed in the embryo prior to sexual differentiation.
3. Fake sex in virgin whiptail lizards - The lizard genus Cnemidophorus reproduces asexually as it only contains females that give birth to clones. Yet they still engage in sexual behavior, with some of the female lizards acting as males would in other genus’s despite the fact such behavior is unnecessary for reproduction.
4. Sexual organs in dandelions - Dandelions have the sexual organs of most flowers, stamens and pistils, yet they also reproduce asexually.
5. Wisdom teeth in humans - Wisdom teeth, a leftover from the good old days when our human ancestors were expected to loose teeth due to a diet of crunchy nuts and tubers, only grow normally in 5% of cases. Originally, the wisdom teeth existed to replace lost molars, but given our current diet and greater oral hygiene, are now a way to keep dentists employed in their removal.
6. Non-functional eyes in blindfish - The species Astyanax mexicanus, known as the blindfish or blind tetra, is naturally found in completely dark caves where eyesight is useless. Yet the blindfish develops eyes in the embryonic stage, only to have them partially reabsorbed. Later the fish grows a flap of skin over the vestigial eye.
7. Human tailbone - covered in point 86
8. Erector pilli and body hair - Humans have a muscle which causes the hair to rise in response to stress (or really good music). This is a vestigial leftover from when our ancestors used such muscles to make their hair rise to appear larger to predators or other threats. Now that humans have too little body hair to appear larger, the muscles are vestigial.
9. Hind leg bones in whales - The existence of embedded leg bones in whales that obviously don’t need them, is vestigial.
10. Wings on flightless birds - Ostriches, Cassowaries, and Kiwis don’t fly, yet they still retain wings that however rudimentary, are still similar in structure to the wings of birds that do fly.
It is also interesting to note that Penguins and Dodos have hollow bones when such hollowness is no longer needed for flight.
These are only a sample of such vestigial organs. In humans there are several further examples of vestigial structures that no longer exist in all individuals. If such structures are so necessary, why are some individuals able to function perfectly well without them? From Edward Willet’s Useless Body Parts at http://www.edwardwillett.com/Columns/uselessbodyparts.htm :
quote:
We have other useless muscles. The extrinsic ear muscles allow many animals to move their ears independently of their heads--all we can manage is a slight wiggle. The subclavius muscle, stretching under the shoulder from the first rib to the collarbone, would be useful if we walked on all fours but is so useless now that some people don’t even have it.
Eleven percent of us also lack the palmaris muscle, a long, narrow muscle that runs from the elbow to the wrist that may have been important when we were tree climbers, and nine percent of us have lost the plantaris muscle, useful for gripping things with your feet. And more than 20 percent of us are missing the pyramidalis muscle, a tiny, triangular pouchlike muscle attached to the pubic bone that may be a relic from the days of our pouched marsupial ancestors.
Other useless body parts include the vomeronasal organ, a tiny pit in the nose lined with nonfunctioning chemoreceptors that may once have been used to sense phermones; the third eyelid, retained as only a tiny fold in the inner corner of the eye, that may once have been a proper eyelid for protecting the eye and sweeping out debris; male nipples (present because milk-producing ducts form in an embryo before sex differentiation); the male uterus (a remnant of an undeveloped uterus that hangs off the male prostate gland); and the female vas deferens, or epoophoron, a cluster of useless dead-end tubules near the ovaries.
The above examples are by no means intended as a comprehensive list, there are at least hundreds of other vestigial structures known to exist among living things, including vestigial code in DNA.
Point 86:
Let’s take a few examples. Like the coccyx, or tailbone. Many evolutionists have claimed that this tailbone is vestigial. But in reality, there are nine muscles attached to it! They help us move, sit, and excrete wastes. Without it, you would have no attachment point for these muscles, and that would not be good!
It is true that there are some muscles attached to the tailbone, which helps explain why this remnant tail still exists. Of course in the embryonic stage, humans do have tails that are reabsorbed, just like blindfish eyes. As other functions ascribed to the coccyx see Talk Reason: arguments against creationism, intelligent design, and religious apologetics :
quote:
The coccyx is a developmental remnant of the embryonic tail that forms in humans and then is degraded and eaten by our immune system (for more detail see the sections on the embryonic human tail and the atavistic human tail). Our internal tail is unnecessary for sitting, walking, and elimination (all of which are functions attributed to the coccyx by many anti-evolutionists). The caudal vertebrae of the coccyx can cause extreme and unnecessary chronic pain in some unfortunate people, a condition called coccydynia. The entire coccyx can be surgically removed without any ill effects (besides surgical complications), with the only complaint, in a small fraction of patients, being that the removal of the coccyx sadly did not remove their pain (Grossovan and Dam 1995; Perkins et al. 2003; Postacchini Massobrio 1983; Ramsey et al. 2003; Shaposhnikov 1997; Wray 1991). Our small, rudimentary, fused caudal vertebrae might have some minor and inessential functions, but these vertebrae are useless for balance and grasping, their usual functions in other mammals.
Point 87:
Another vestigial organ that many evolutionists have listed is the appendix. So they go by that idea and remove your appendix if it is diseased, just like they did with the tonsils. But the appendix is actually a part of our immune system. It helps in the fighting of infections in the intestines.
If the human appendix is so vital for survival, why can people function without it? Any apparent advantage in retaining this structure is more than offset in cases of appendicitis, which often result in death if the appendix is not removed.
Again fro the Live Science site:
quote:
In 2000, in fact, there were nearly 300,000 appendectomies performed in the United States, and 371 deaths from appendicitis. Any secondary function that the appendix might perform certainly is not missed in those who had it removed before it might have ruptured.
Point 88:
At one point in history, evolutionists thought that the human embryo had vestigial organs. But each of these has been shown to have important functions in our body.
Each ”organ’ or more properly structure in a human embryo has an important function in the human body? Like the tail in the human embryo which does not normally protrude from the body? What of gill slits which in fish become gills but in humans become, among other things, parts that become the human ear. Yes embryonic structures do develop in embryos that diverge into different functions according to the evolution of the host. From 29+ Evidences for Macroevolution: Part 2 :
quote:
There are numerous other examples in which an organism's evolutionary history is represented temporarily in its development. Early in development, mammalian embryos temporarily have pharyngeal pouches, which are morphologically indistinguishable from aquatic vertebrate gill pouches (Gilbert 1997, pp. 380, 382). This evolutionary relic reflects the fact that mammalian ancestors were once aquatic gill-breathing vertebrates. The pharyngeal pouches of modern fish embryos eventually become perforated to form gills. Mammalian pharyngeal pouches of course do not develop into gills, but rather give rise to structures that evolved from gills, such as the eustachian tube, middle ear, tonsils, parathyroid, and thymus (Kardong 2002, pp. 52, 504, 581). The arches between the gills, called branchial arches, were present in jawless fish and some of these branchial arches later evolved into the bones of the jaw, and, eventually, into the bones of the inner ear as recounted above and in prediction 1.4, example 2.
Point 89:
Now let’s examine the reasoning behind vestigial organs. If we indeed do have vestigial organs today, then that means they had to have been functioning in our supposed ancestors. But that means we are degrading, losing the functioning of our organs.
No, it means that some organs are unnecessary baggage from our ancestors, not that all organs are ”degrading.’ It just represents more evidence of evolution.
Point 90:
But evolution requires just the opposite, upgrading, from a single cell to a human. So are we upgrading, degrading, or neither?
Evolution only explains what is and how it got here. It does not make moralistic judgments concerning upgrading, degrading, or either. As far as evolution is concerned, all organisms that exist today are equally adapted to their environment and therefore equally evolved.
The idea of ”degrading’ in criticism of science is usually reserved for fundamentalist religious cults where life, including human life, is supposed to be less ”perfect’ as time goes on. Considering the message of self-hate common in such cults, in my opinion the term ”degrading’ is appropriate in more ways than one.

Replies to this message:
 Message 87 by Someone who cares, posted 12-26-2006 8:53 PM anglagard has not replied
 Message 88 by Someone who cares, posted 01-11-2007 1:10 AM anglagard has not replied
 Message 89 by Someone who cares, posted 01-24-2007 10:41 PM anglagard has not replied

  
anglagard
Member (Idle past 836 days)
Posts: 2339
From: Socorro, New Mexico USA
Joined: 03-18-2006


Message 79 of 100 (365886)
11-24-2006 11:04 PM


Radiometric Dating
Point 91:
Next, we will examine the dating methods. Many evolutionists use certain dating methods to tell the age of fossils, and come up with results just like their theory would propose, some ages around millions of years old or more. Let’s examine how valid these dating methods really are.
The dates are not “results just like their theory would propose.” To understand how dating methods have evolved, a history lesson is in order. It was obvious to anyone who seriously studied geology that the Earth was older than 6000 years old and that there was no evidence for a global flood before Darwin’s Origin of Species was published in 1859. From Changing Views of the History of the Earth
quote:
1774 Comte de Buffon: Epochs of Nature. Buffon assumed that the earth started molten, measured cooling rates of iron spheres, scaled up, and calculated the age at ~75,000 years. He himself was suspicious that this was much too young and, in manuscripts published after his death, suggested longer chronologies, including one estimate of nearly 3 billion years.
quote:
1788 James Hutton: Theory of the Earth; or, an investigation of the laws observable in the composition, dissolution and restoration of land upon the globe. Hutton is traditionally credited with being the father of modern geology. He was the first modern uniformitarian. Hutton argued that the Earth was of immense antiquity, cycling through changes via slow processes sans catastrophes. The last sentence of Hutton's 1788 work is famous and is widely quoted:
The result, therefore, of our present enquiry is, that we find no vestige of a beginning - no prospect of an end.
quote:
1794 Robert Townson: Philosophy of Mineralogy. Townson was one of the many catastrophists of the late 18'th and early 19'th century. He pointed out that fieldwork had revealed that the features of the surface of the Earth could not be accounted for by a single Creation and catastrophic flood but rather successions of formation and dramatic change.
quote:
1857 Hugh Miller: The Testimony of the Rocks. Miller was a very popular creationist geologist. He believed that the Noachian flood was a local flood in the Mideast and did not credit the theory that the Earth was young. On page 324 he wrote:
"No man acquainted with the general outlines of Palaeontology, or the true succession of the sedimentary formations, has been able to believe, during the last half century, that any proof of a general deluge can be derived from the *older* geologic systems, -- Palaeozoic, Secondary [Mesozoic], or Tertiary."
Obviously, such ideas concerning an ancient Earth that predate Darwin’s Origins falsifies your apparent assertion that evidence from geology is used to fit into the concept of Darwinian evolution. Geoscientists, being scientists, use evidence for their hypothesis. They rarely, if ever, conspire to falsify data, as YEC ”scientists’ often do since there are undesirable consequences to telling lies in real science.
Continuing from Darwin in 1859 to the discovery of radioactivity in 1905.
quote:
1862 Lord Kelvin: On the Secular Cooling of the Earth. Using thermodynamic principles and measurements of thermal conductivity of rocks, Kelvin calculated that the earth consolidated from a molten state 98 million years ago. In 1897, he revised his estimate to 20-40 million years. Dalrymple says that Kelvin's estimates were "highly authoritative" for three decades, but notes that they were challenged by people from several fields, including T. H. Huxley, John Perry (a physicist), and T. C. Chamberlain (a geologist).
quote:
1893 Charles D. Walcott: Geologic Time, as Indicated by the Sedimentary Rocks of North America. Walcott takes a detailed look at the Paleozoic sediments of the Cordilleran Sea (just east of the Sierra Nevadas), considering such things as the land area supplying sediments and the grain sizes of the sediments. He arrived at an estimate of 17.5 million years for the Paleozoic and, based on various other authors' estimates of relative ages of the other eras, 55 million years for the earth.
As can be seen several individuals that had actually studied the evidence long before the discovery of radioactivity knew the Earth was far older than any 6000 years and that there was no evidence for any global flood 4500 years ago.
Point 92:
One rock, found in Nigeria, was dated to be 30 million years old with the dating method - fission track dating. Then, this same rock, was dated to be 95 million years old with the dating method of Potassium-Argon dating. Yet with Uranium-Helium dating, this same rock was dated to be 750 million years old! [36]
Because the article referred to in this statement is not readily available, I must await the fulfillment of an interlibrary loan in order to examine the actual circumstances and full content of this assertion. Rest assured, because of the possibly unintentional misrepresentation of your source in the essay covered under point 83, I will be examining all of your citations.
Point 93:
Using carbon-14 dating, a freshly killed seal was dated to have died 1,300 years ago! [37]
Here is why, from CD011.4: C-14 age of a seal
quote:
This claim derives from Wakefield (1971):
Radiocarbon analysis of specimens obtained from mummified seals in southern Victoria Land has yielded ages ranging from 615 to 4,600 years. However, Antarctica sea water has significantly lower carbon-14 activity than that accepted as the world standard. Therefore, radiocarbon dating of marine organisms yields apparent ages that are older than true ages, but by an unknown and possibly variable amount. Therefore, the several radiocarbon ages determined for the mummified seal carcasses cannot be accepted as correct. For example, the apparent radiocarbon age of the Lake Bonney seal known to have been dead no more than a few weeks was determined to be 615 +/- 100 years. A seal freshly killed at McMurdo had an apparent age of 1,300 years.
(the full citation is: Wakefield, Dort, Jr., 1971. Mummified seals of southern Victoria Land. Antarctic Journal 6(5): 210-211.)
This is the well-known reservoir effect that occurs also with mollusks and other animals that live in the water. It happens when "old" carbon is introduced into the water. In the above case of the seal, old carbon dioxide is present within deep ocean bottom water that has been circulating through the ocean for thousands of years before upwelling along the Antarctic coast.
Point 94:
Also using carbon-14 dating, presently living mollusk shells were dated to be 2,300 years old! [38]
The cited article is about scientists discovering anomalous dates and trying to evaluate how such anomalous dates occur according to real science, not ”creation science.’ This problem has been examined and explained.
From How Good are those Young-Earth Arguments: Radiocarbon Dating
quote:
Since limestone is very old it contains very little carbon-14. Thus, in getting some of their carbon from limestone, these mollusks "inherit" some of the limestone's old age! That is, the limestone carbon skews the normal ratio between C-12 and C-14 found in living things. No problem! If one dates such mollusks, one must be extra careful in interpreting the data. Not every mollusk shell presents such problems, and the dating of other material might yield a cross-check. Further study might even allow correction tables. The discovery has strengthened the carbon-14 method, not weakened it! By the way, shouldn't the creationist be worried over the old, carbon-14 age of the limestone? Why is it that limestone has so little C-14 in it?
That last sentence of the quote is a very good question for YECs. Doesn’t the lack of C14 in the limestone indicate it is older than any flood?
Point 95:
Using carbon-14 dating as well, the shells of living snails were dated to have died 27,000 years ago!!! [39]
The cited article is about scientists discovering anomalous dates and trying to evaluate how such anomalous dates occur according to real science, not ”creation science.’ As mentioned above, there are certain specific conditions where radiocarbon dating is not accurate such as in environments where mollusks (snails) show false dates of greater age than any 4500 year old flood or Garden of Eden. From : How Good are those Young-Earth Arguments: Additional Topics
quote:
Either we have a worldwide conspiracy among geologists, which no sane person believes, or else the numerous radiometric dates were consistent enough to allow that kind of close agreement. In fact, Dr. Dalrymple, an expert in radiometric dating with lots of hands-on experience, puts the percentage of bad dates at only 5-10 percent.
Thus, we clear away the first illusion spun by creationism, namely that most of the dates are bad, that the radiometric picture is totally chaotic. In fact, it is not at all unusual for several different radiometric methods to agree within a few percentage points on a date. When you consider that each radiometric method is subject to different types of error, that the different "clocks" run at different speeds, such an agreement would be extremely rare on the basis of pure chance. In a number of instances, more than you might imagine, dates are further corroborated by methods that have nothing to do with radioactivity. Thus, the big, statistical picture painted by radiometric dating is excellent. Today, we have some 100,000 radiometric dates, the vast majority contributing sensibly to the overall picture.
Point 96:
What absurdity! See also, that the various dating methods give various ages to the same fossil or geologic layer. Using this, an evolutionist can use the method he desires that would give him the age he desires, when another test would show the specimen being dated to be millions of years older or younger!
Instances of radioisotope dating where apparent results conflict with other forms of evidence are studied by real scientists, as opposed to universally condemned by “creation scientists.” Through a thorough study, such scientists come up with explanations of the few anomalous findings out of the over 100,000 dates measured so far. One or two dates that do not fit in merit examination according to real science, whereas under ”creation science’ one thing that is anomalous is seen to destroy the entire basis of physics, geoscience, and bioscience. However such criticism of radiocarbon dating due to actually discovering a few examples where the dates given are wrong due to processes that are well understood does nothing to invalidate all radioisotope dating, including radiocarbon dating in the vast majority of cases where such dating is highly correlated by using several methods, several samples, and several laboratories. For one example, consider this recent post by Mr Jack concerning the K-T boundary at Message 29
quote:
Carbon dating is not used; these methods are:
quote:
________________________________________
There we find the following data for the Z-coal strata of the Hell Creek Formation presented in the order of; Material, Method, # of samples, Result in Millions of Years
tektites, 40Ar/39Ar total fusion, 28, 64.8 ++ 0.1
tektites, 40Ar/39Ar age spectrum, 1, 66.0++0.5
tektites, 40Ar/39Ar age spectrum, 1, 64.7++0.1
tektites, 40Ar/39Ar total fusion, 17, 64.8++0.2
biotite & sanidine, K-Ar, 12, 64.6++1.0
biotite & sanidine, Rb-Sr isochron (26 D.P.), 1, 63.7++0.6
zircon, U-Pb concordia (16 data), 1, 63.9++0.8
________________________________________
- from Radiometeric Dating Does Work! by G. Brent Dalrymple.
Why do these different methods produce consistent results?
A question many here, including RAZD, Mr Jack, many others, and I would like to see answered, if it’s not too much trouble.
Point 97:
Plus, carbon-14, as well as some other radioactive dating methods, rely on unprovable assumptions about the rate of decay, the environment condition and others - things we cannot know for certain if they were truly this way. This is why the dating methods are inconsistent and false.
The dating methods, when properly used, are amazingly consistent with other dating methods, which is why it is difficult to just call all examples false without adequate thought or explanation. The physical laws concerning radioactive decay and ratios of stable isotopes (used to determine conditions in past environments) would all have to be manipulated in different ways to produce the same entirely consistent false result.
Here is a list of dating methods that have been correlated down to an annual basis from my favorite thread on EvC, coincidently created by RAZD at Message 1
quote:
For anybody unclear on the concept, this is how it stacks up -- the minimum age of the earth is:
8,000 years by annual tree rings from Bristlecone pine in California.
10,000 years by annual tree rings from Oaks in Europe (different environment and location)
45,000 years by annual varve layers of diatoms in Lake Suigetsu, Japan (different biology and location)
” ... corroborated by Carbon 14 (C-14) radiometric dating (limit 50,000 years by half life)
110,000 years by annual layers of ice in Greenland (different process altogether)
422,776 years by annual layers of ice in Antarctica (different location altogether)
567,700 years by annual layers of calcite in Devil's Hole (another different process and location altogether)
” ... corroborated by Thorium-230 dates and Protactinium-231 radiometric dating (independent processes)
” Even greater age implied by daily layers of coral (another different biology, process and location, again)
” ... some additional information including some cool slideshow websites
Anyone interested in debunking the science behind radiometric dating must understand the content of this thread, either through actually reading it or by reading similar arguments from the literature of real science, which is overwhelmingly supported by evidence. The above are dating methods that are essentially accurate to a year by year basis.
As I noted before, the position explained in this thread has never been refuted. Not by AIG, ICR, Hovind (currently imprisoned for suspicion of tax evasion), or by any of their followers who endlessly regurgitate, intentionally or otherwise, the bearing of false witness (or in some cases theft) without thought or consideration for any eternal judgment based upon the 10 Commandments.
In addition to the methods of radiometric dating outlined above there is also considerable agreement with other non-annual dating methods such as paleomagnetism, plate tectonics, the fossil record, and of course other various methods of radiometric dating such as:
Samarium-147/Neodymium-143 -half life 106 billion years; Rubidium-87/Strontium-87 - half life 48.8 billion years; Rhenium-187/Osmium-187 - half life 42 billion years; Lutetium-176/Hafnium-176 - half life 38 billion years; Thorium-232/Lead-208 - half life 14 billion years; Uranium-238/Lead-206 - half life 4.5 billion years; Potassium-40/Argon-40 - half life 1.26 billion years; Uranium-235/Lead-207 - half life 0.7 billion years; Beryllium-10/Boron-10 - half life 1.52 million years; Chlorine-36/Argon-36 - half life 300,000 years; Carbon-14/Nitrogen-14 - half life 5715 years; Uranium-234/Thorium-230 - half life 248,000 years; Thorium-230/Radium-226 - half life 75,400 years.
See the source for the above at Radiometric Dating A Christian Perspective at http://www.asa3.org/aSA/resources/Wiens.html by Dr. Roger C. Wiens.
Point 98:
But there is also circular reasoning involved in dating geologic layers and fossils. When the scientists are asked to tell the age of a certain fossil, they check the date for the geologic layer in which the fossil was found, and give that age to the fossil. Then, if asked to tell the age of a certain geologic layer, they look at the dates of the index fossils, found within that geologic layer, and give that age to the geologic layer! This is circular reasoning!
The relative geologic timescale was worked out before Darwin. See http://www.ncseweb.org/..._in_the_grand_canyo_12_30_1899.asp
quote:
A major omission in MTC (added for clarity - Grand Canyon: Monument to Catastrophe (MTC)) is any discussion of the great synthesis of worldwide geologic observations known as the Geologic Time Scale. Wise (1998) pointed out that the creationist time scale ignores the countless worldwide geological studies which show that, on all continents, the same general sequence of sedimentary rocks occurs, and that the major and minor divisions in this sequence are characterized by the specific assemblages of fossils they contain. This overall scheme was essentially completed before 1859 when Darwin published his On the Origin of Species by Means of Natural Selection. No assumptions of organic evolution were made in deriving the geologic column or in using fossils for correlation of strata from continent to continent. However, the eras and periods of the geologic time scale, with which we divide geologic time, reflect the dramatic changes which have occurred in the history of life on earth recorded in the rocks (Gould 1994). Because of these changes, the fossil assemblages found in each geologic system are distinct, permitting us to make worldwide stratigraphic correlations. Today such correlations are also made using non-paleontological criteria, such as radiometric dating, and sequences of magnetic reversals and of light stable isotope ratios, particularly carbon isotopes (Bowring and Erwin 1998).
Evidently you are unaware of other collaborating dating evidence such as the additional use of paleomagnetic data in determining the age of geologic formations. Light stable isotope ratios, which are used to determine past climates and ecological conditions are also a different and collaborating field from radiometric dating. See Stable Isotopes for Dummies at Archaeology if unclear on the concept.

Replies to this message:
 Message 90 by Someone who cares, posted 01-25-2007 12:04 AM anglagard has not replied

  
anglagard
Member (Idle past 836 days)
Posts: 2339
From: Socorro, New Mexico USA
Joined: 03-18-2006


Message 83 of 100 (367510)
12-02-2006 6:52 PM


Supposed Loss of Genetic Information
Point 99:
In addition to all this, evolutionists often claim that evolution or its processes has been observed in one or another example. Let us examine what was really observed in these examples. One popular example of natural selection is the pepper moth. With the photo of a white one sitting on a tree and a black one sitting on a tree. The claim is that the white pepper moths and black ones survived by natural selection as the tree color changed from white to black, due to pollution and smog. But this is not an example of natural selection working for macroevolution! This is an example of natural selection in two types of moths, not showing how natural selection would work to make macroevolution any more plausible. What we have here is the change of amounts of different colored moths, from white to black, in order to camouflage and not be seen by predators.
The example shows natural selection, as claimed by you and supporters of evolution. Since there is no disagreement, what’s the point?
Point 100:
No new information was added to the genetic code, as evolution would require in evolving a single cell to a human over time. Also, the pepper moths stayed pepper moths, they didn’t evolve into, say, butterflies, or any other creatures of different kinds, but that is what would really be natural selection for evolution in action. Another example often used is wingless beetles. Because on an island, these beetles wouldn’t be blown off by the strong winds. So in this case, the mutation’s effect was beneficial. But, the mutation itself was still a defect, it was a loss, a loss of code for wings. This was a negative mutation, it removed information, it didn’t add any new information to the genetic code, as evolution would require. So this example does not show evolution or help it.
I take it this is about gain or loss of information. If the information necessary to make the majority of moths darker or lighter is still present, yet one or the other is favored by the environment, how is information lost? When the environment changes back to non-polluting, the majority of moths return to a lighter shade. Where is that loss of information if the moths go from light to dark then return to light? If the moths went from light to dark, and then were unable to return to light, that would show this loss of information, not the ability to become light or dark as circumstances permit.
Assuming a loss of information in any reversible change is easily refuted by the application of simple logic. From: SLOT88 Situs Judi Slot Online Terpercaya No 1 di Indonesia
quote:
Perhaps the most remarkable aspect of this ID claim is that it can be refuted from first principles, without needing any specialized knowledge or evidence! Suppose there are two nucleotide sequences A and B. If some particular mutation X can transform sequence A into sequence B, there is another particular mutation Y which can transform sequence B into sequence A. If mutation X is one which subtracts information from a sequence, it follows that sequence A must contain more information than sequence B -- and mutation Y must, therefore, be one which adds information to a sequence.
Point 101:
Yet another example often brought up is viruses that have adapted from harming one animal type to harming humans. But even this is not evolution. Why? Because no new information was added to the genetic code, and because the virus stayed a virus. Yes, it may have mutated to do harm in another creature or humans, but no new code was added; and the virus didn’t evolve into, say, a bacteria or any other organism; the virus stayed a virus. This is why viruses don’t show evolution. Same with AIDS or HIV, not evolution; for the same reasons as with the virus. Also, with antibiotic resistance, we observe something similar. Loss of information may have caused the antibiotic resistance in a certain organism, but this was a loss of information, not a gain, as evolution would require. Never has it been observed in nature that new information was added to the genetic code of an organism, only loss. So evolution has not been observed in any of these or other examples.
What constitutes “loss of information,” do you know?
According to CB102: Mutations adding information concerning any loss of information in evolution:
quote:
It is hard to understand how anyone could make this claim, since anything mutations can do, mutations can undo. Some mutations add information to a genome; some subtract it. Creationists get by with this claim only by leaving the term "information" undefined, impossibly vague, or constantly shifting. By any reasonable definition, increases in information have been observed to evolve. We have observed the evolution of
” increased genetic variety in a population (Lenski 1995; Lenski et al. 1991)
” increased genetic material (Alves et al. 2001; Brown et al. 1998; Hughes and Friedman 2003; Lynch and Conery 2000; Ohta 2003)
” novel genetic material (Knox et al. 1996; Park et al. 1996)
” novel genetically-regulated abilities (Prijambada et al. 1995)
If these do not qualify as information, then nothing about information is relevant to evolution in the first place.
There are many more examples of increased genetic information being noted through actually observing nature. The concept of genetic information always being lost has also been argued here several times without success, most recently starting here Message 89 and here Message 69. In both cases you will notice the creationist side has had to concede that loss of information is not true in all cases. Therefore your statement “Never has it been observed in nature that new information was added to the genetic code of an organism, only loss” is not supported even by your fellow travellers.
The increase in information in any code subject to a process of natural selection and mutation has even been shown using computer simulations. From the abstract of Just a moment... :
quote:
To make a case for or against a trend in the evolution of complexity in biological evolution, complexity needs to be both rigorously defined and measurable. A recent information-theoretic (but intuitively evident) definition identifies genomic complexity with the amount of information a sequence stores about its environment. We investigate the evolution of genomic complexity in populations of digital organisms and monitor in detail the evolutionary transitions that increase complexity. We show that, because natural selection forces genomes to behave as a natural "Maxwell Demon," within a fixed environment, genomic complexity is forced to increase.
From the top two examples alone, one can see the concept of loss of information as invalidating evolution is shown to be false by observing nature and by simulating nature in the lab. I think this argument is similar to the “no transitional fossil” argument since any example of transitional fossils or increase of genetic information will simply be handwaved away by redefining ”transitional fossil’ or ”information.’ However, creating a false definition of either concept does nothing to invalidate the real definition of the concept, it only invalidates the false definition.
Therefore the concluding assertions presented in this paragraph of the essay are either false or meaningless.

Replies to this message:
 Message 93 by Someone who cares, posted 02-15-2007 12:11 AM anglagard has not replied

  
anglagard
Member (Idle past 836 days)
Posts: 2339
From: Socorro, New Mexico USA
Joined: 03-18-2006


Message 84 of 100 (367893)
12-06-2006 2:28 AM


YEC Ecology of Unnecessary Death and Disease
Point 102:
Now we will examine a few more difficulties with evolution. Life has many interdependent cycles. Each cycle relies heavily on its own parts working together, and on other cycles functioning with it. There is a perfect harmony in nature, and many intelligently made cycles exist.
It appears to me that a lot of species are disappearing in nature, in fact have disappeared in the past, how is that perfect harmony? Nature seems more a struggle to survive than some example of perfect harmony.
Point 103:
Take for example the water cycle: evaporation, condensation, and precipitation. It works out very well and even recycles, to an extent, our water. How could random, chance, unguided processes do this?
The water on the Earth is not recycled to an extent, it is, with the very minor exception of additional inputs/outputs to/from space, recycled completely. What is meant by random or chance by evaporation, condensation, and precipitation? How this works is well understood by using physics. All physical systems (as opposed to biological systems) are in overall balance, what goes in one end must come out the other, what is so magical about that?
Point 104:
Or take ecology. There is a relationship between animals and their environments, and the climates. All of this is connected, and if you remove or change one, it may affect the whole cycle, and it may end in the extinction of certain animals. Or take the food chain. A very close relationship exists here. Remove one species of animals, and the effect can be devastating! Everything is so dependent upon something else. How could evolution do this?
Easily, by using an immense amount of time to simultaneously evolve. Independent parts of a system can simultaneously change and adapt as is seen everyday. I must admit that I am puzzled by your argument that evolution is false because two different things can’t happen at the same time, such as one organ coevolving with the other. Two, and more, different things happen simultaneously everyday.
Point 105:
Yet another problem. If the time dates of evolutionists are true, the world would be overflowing with humans now. But that’s not the case.
The term “overflowing with humans” is at best subjective considering that there are 6.67 billion people on the planet and that overpopulation is considered a problem. This is because births and living to reproduce far outnumbers deaths. In history this was not always the case, are you arguing the opposite is true, that number of births that live to reproduce has always significantly exceeded number of deaths in human history prior to the Industrial Revolution? All the population curves in history I have seen look like this:
Notice that the world population did not significantly increase until YEC medievalism gave way to science, and consequently the Industrial Revolution, as the prevailing paradigm for creating food, medicine, technology, and therefore an increased standard of living.
It is true however that under a “creation science” only program, I’m sure the population could be significantly reduced through creationist biology to kill in the hospital and, along with creationist geology, reduce the collective standard of living. It would be just like having population growth control Aztec style, by killing in order to appease a false YEC god, without the gore or the guts.
There may be several species that would be in favor of your desire to destroy real science and thereby reduce the human population. However, such a stand may be underappreciated by those members of the human species who would be condemned to an early death.
Point 106:
Also, according to the studies of biochemists Allen Wilson and Vincent Sarich, the first people had to have come originally from only two people, and had to appear less that 200,000 years ago. [40] Creation fits this description perfectly; whereas evolution cannot speak.
Do you have any idea how Wilson and Sarich came to this conclusion?
From : http://www.jqjacobs.net/anthro/paleo/genome.html
quote:
In 1967 Vincent M. Sarich measured the evolutionary distance between humans and chimpanzees by studying their blood proteins. The accumulated differences reflect mutations since species divergence. His findings refined the genetic distance, dating it to between five and seven million years ago, compared to a previously estimated 15 million. That work used blood proteins. Since the 1980's we are able to sequence DNA. Wilson and Cann studied mitochondrial DNA (mtDNA).
And after Sarich died, but using his methods for further refinement:
quote:
Wilson and Cann calculated how much humans had diverged from one another relative to how much they had diverged from chimpanzees, and determined the ratio was less than 1:25. Assuming five million years since human/chimp divergence results in an estimate of 200,000 years to our common maternal ancestor. They also measured how much the mtDNA has evolved in the aboriginal populations of New Guinea and Australia. The result, about one third that of the species, infers coalescence 150,000 to 180,000 years ago given presently understood settlement dates of those areas.
So by using assumptions from evolutionary theory to come to the conclusion modern humans had a common ancestor some 200,000 years ago means “Creation fits this description perfectly; whereas evolution cannot speak.” Do you notice any irony here?
In case you don’t see the irony, did you notice that you are quoting as support for your essay research that totally invalidates the story of Noah’s Ark?
Point 107:
Look at nature. Look at the animals, the plants. Look at the symmetry of a flower. Look at yourself, marvel at how you can balance yourself and walk upright. Look at your hand, think of all the actions you can perform, think how precisely you can control it to put a thread through the eye of a needle. Think about your brain, the many tasks you can do, and what you can control with your brain. Was all of this a product of mere chance? Of unguided, random processes?
Natural selection is not a random process. Genetic mutation alone is proof against some continuously monitored intelligent design by a benign deity unless the deity has a sick sense of humor. Please tell our audience what the characteristics of a continuously intrusive intelligent designer would be that did this:
Thanks goes to Neil deGrasse Tyson for this argument in the video Incompetent Design which, among other arguments that work against this paragraph of the essay, can be seen here:
http://youtube.com/watch?v=61hfTFZ9ST8

Replies to this message:
 Message 94 by Someone who cares, posted 02-15-2007 12:43 AM anglagard has replied

  
anglagard
Member (Idle past 836 days)
Posts: 2339
From: Socorro, New Mexico USA
Joined: 03-18-2006


Message 91 of 100 (379660)
01-25-2007 12:18 AM
Reply to: Message 60 by Someone who cares
10-19-2006 11:39 PM


Catching Up & Etc.
S1WC writes:
As for your essay, it should be interesting. I hope that when you write your essay and post it on the world wide web you will understand how I felt writing mine and having others constantly attack it. I don't really mind debating material where I was really mistaken, it helps me learn, but I feel that some of the other more controversial debating where we can't get anywhere is excessive. Maybe you'll understand this when others take apart your essay, one possible member being me... To make it fair.
Just thought I'd let you know I'm still here and that I will soon post as you are almost caught up. Also, in regard to my religious beliefs, they are beginning to be discussed here at Message 1 should you choose to debate these as well someday when you have the time.

This message is a reply to:
 Message 60 by Someone who cares, posted 10-19-2006 11:39 PM Someone who cares has replied

Replies to this message:
 Message 92 by Someone who cares, posted 02-14-2007 11:10 PM anglagard has not replied

  
Newer Topic | Older Topic
Jump to:


Copyright 2001-2023 by EvC Forum, All Rights Reserved

™ Version 4.2
Innovative software from Qwixotic © 2024